You are on page 1of 59

Study Guide Infection and Infectious Diseases

CONTENTS

Table of contents ...

Introduction .

Curriculum

Block Team ...................

Facilitators

Time Table Regular Class

10

Time Table English Class ..

10

Meeting of Students representatives

20

Meeting of The Facilitators.

20

Assessment method ...

20

Learning Program ..

25

Lecture .

25

Learning Task and Self Assessment. 25

Curriculum Mapping ...

57

Reference

58

Faculty of Medicine UNUD,MEU

Study Guide Infection and Infectious Diseases

INTRODUCTION
Due to the application of integrated curriculum at the Faculty of Medicine Udayana
University, the discipline-based subjects of the previous curriculum such as Biology,
Anatomy, Physiology, Internal Medicine, etc have been integrated and incorporated into
several blocks. One of these blocks is Infections and Infectious Diseases. In this block will
be explained in general about pathogenesis, pathophysiology, sign, symptoms, clinical
features, diagnosis, and management of certain infectious diseases commonly occur in
community.
This guide book aims to give general information for medical students about infections
and infectious diseases and important for facilitators and resource person while facilitate or
guiding the students in learning process. This study guide consists of general information on
learning time table, block team members, facilitators, and the core curriculum including
learning outcomes, learning situations, learning tasks and self-evaluation items.
The block Infection and Infectious Diseases has the equivalent of (six) credits. As a
block of six credits, the learning processes will be carried out for 30 days starts from 27th of
November 2013 as shown in the Time Table. The final examination will be conducted on 13th
of Junuari 2014. During the 30 days of learning activities, the students will discuss several
topics in varied forms of learning situations such as independent learning, small group
discussion, lecture, and skill lab.
More than half of the learning material must be learned independently and in small
group discussions. A lecture is given only to emphasize crucial things or objectives of
material and to prepare the students before discussion. For small group discussion, the
students will be given learning tasks to solve and discuss. After discussion, students also
have to evaluate their learning progress independently (self evaluation).
From this block, we hope every medical student have knowledge and skill to diagnose
and manage infections and certain infectious diseases commonly occur in community, as a
frontline in community health.
Since the integrated curriculum of the Faculty of Medicine Udayana University is still in
progress, this Study Guide will also, naturally, have some revisions in the future. Therefore,
we kindly invite readers to give any comments or suggestions for its improvement and
development.

Planners

Faculty of Medicine UNUD,MEU

Study Guide Infection and Infectious Diseases

CURRICULUM OF THE BLOCK


AIMS
To comprehend the biology of the infectious diseases
To apply and interpret common laboratory diagnosis of infectious diseases
To diagnose and manage common infectious diseases
To carry out basic immunization in children and adults
LEARNING OUTCOMES

Comprehend the practical and clinical implications of the biology of infection

Apply the general principles of approach to patients with infectious diseases

Apply and interpret common laboratory diagnosis of common infectious


diseases

Apply the basic principles of immunization in children and adults

Diagnose and manage common bacterial infections (common Gram positive


and negative, spirochetal)

Diagnose and manage common parasitic infections (common nematode,


trematode, cestode, and protozoal infections)

Diagnose and manage common fungal infections


Clinically diagnose and manage common viral infections (caused by common
respiratory virus, herpesvirus, arbovirus)
Clinically diagnose and manage puerperial Infection

CURRICULUM CONTENT
1. The biology of infection: bacterial, viral, fungal and parasitic.
a. Principles of bacterial infections such as Staphylococci, Streptococci,
Neisseria, Salmonella, Vibrio, anaerobic bacteria Leptospira, Mycobacteria,
Gram positive bacilli)
b. Principles of viral infections such as respiratory virus (influenza virus,
mumps, measles), retrovirus (HIV), herpesvirus (HSV 1, HSV 2, VZV,
arbovirus (dengue virus, Japanese B encephalitis virus).
c. Principles of fungal infections such as Candida, Pneumocytis jiroveci,
Histoplasma, Cryptococcus
d. Principles of parasitic infections such as Plasmodium, Toxoplasma gondii,
Entamoeba histolytica and soil transmitted helminthes.
2. General approach to the patients with infection such as:
a. Clinical manifestations (local and systemic infections)
b. Laboratory examination to support diagnosis of infections i.e. Microbiological
examination,
Parasites examination, Clinical pathology examination,
Pathology examination and Imaging examination

Faculty of Medicine UNUD,MEU

Study Guide Infection and Infectious Diseases

3. Management patients with infection such as:


a. Common bacterial infections such as bacterial meningitis, typhoid fever,
diarrhea, endocarditis, diphtheria, tetanus, food poisoning, genital
gonorrhoeae, non gonococcal urethritis, etc.
b. Common parasitic infections such as malaria, amoebiasis, toxoplasmosis.
c. Common fungal infection such as dermatophytosis, systemic candidiasis,
histoplasmosis, cryptococcosis, pneumocytis jiroveci pneumonia.
d. Common viral infections such as mumps, measles, influenza (especially
H5N1), SARS, varicella, herpes labialis, herpes genitalis, dengue fever,
Japanese B encephalitis, and HIV.
4. Immunization in children and adults, and general advice to international traveler
5. Puerperial Infection

Faculty of Medicine UNUD,MEU

Study Guide Infection and Infectious Diseases

STANDAR KOMPETENSI DOKTER

Faculty of Medicine UNUD,MEU

Study Guide Infection and Infectious Diseases

Faculty of Medicine UNUD,MEU

Study Guide Infection and Infectious Diseases

PLANNERS TEAM
No
1

Name

Departement

Prof. Dr. dr. Tuti Parwati Merati, SpPD,


KPTI (Coordinator)

Internal Medicine

Phone
08123806626

Dr. dr. Bagus Komang Satriyasa,


M.Repro (Secretary)
Dr. dr. Dewa Made Sukrama, M.Si,
SpMK
Prof. Dr. dr. Raka Sudewi, Sp.S (K)

dr. IGK Darmada, SpKK

dr. I.B. Ngurah, M.For

Pharmacology

08123687288

dr. Agus Somia, Sp.PD

Internal Medicine

08123989353

dr. Made Sudarmaja, M.Kes

Parasitology

08123953945

2
3

Pharmacology

081237166686

Microbiology

081338291965

Neurology

0816710244

Dermatology and
Venereology

081338044921

LECTURER
NO

2
3
4

NAME
Prof. Dr. dr. Tuti Parwati Merati, SpPD,
KPTI
Prof.Dr. dr. Raka Sudewi, Sp.S (K)
Prof.Dr.dr. I.B. Rai, SpP (K)
dr. Agus Somia, SpPD

dr. A.A.G.P. Wiraguna, SpKK

Prof.dr. M. Swastika Adiguna, SpKK (K)

dr. IGA. Sumedha Pindha, SpKK (K)

10

dr. Dwi Lingga, SpA (K)/dr W.


Gustawan,M.Sc., Sp.A
dr. IGK. Darmada, SpKK/ dr. Darmaputra,
Sp.KK
dr. Ni Made Aditarini Sp. MK

11
12

dr. Luh Ariwati


dr. Nyoman Mahartini, SpPK

13

8
9

DEPT
Internal Medicine
Neurology
Pulmonology
Internal Medicine
Dermatology &
Venereology
Dermatology &
Venereology
Dermatology &
Venereology
Child Health
Dermatology &
Venereology
Microbiology

PHONE
08123806626
0816710244
08123804579
08123989353
081338645288
08123828548
08155735977
08125684656/
08123848241
081338044921

Parasitology
Clinical Pathology

081338675344
08123662311
081337165577

dr. I.B. Ngurah, M. For

Pharmacology

08123687288

14

Dr. dr. Bagus Komang Satriyasa, M.Repro

Pharmacology

081237166686

15

Parasitology

08124649002

Parasitology

08113804500

17

dr. Kadek Swastika, M.Kes


Prof. dr.Dewa Putu Widjana, DAP&E,
Sp.ParK.
Prof. dr. IGM. Aman, SpFK

Pharmacology

081338770650

18

dr. Sri Budayanti, Sp.MK

Microbiology

08583711398

19

dr.Dewa Ayu A. Sri Laksmi,M.Sc


dr. Made Susila Utama, Sp.PD

Parasitology
Internal Medicine

081392017107

16

20

Faculty of Medicine UNUD,MEU

08123815025

Study Guide Infection and Infectious Diseases

21

dr.Made Agus Hendrayana, M.Ked

22

24

dr.Lely Rahayu, Sp.THT-KL


dr. I Gusti Made Surya Candra Trapika,
M.Sc
dr. K. Januartha, M.Kes

25

dr. Made Jawi, M.Kes

26

dr. I Nyoman Bayu Mahendra,Sp.OG

27

I.B. Nyoman Putra Dwija, S.Si, M.Biotech

28
29

23

Microbiology

08123921590

ENT

08113809882

Pharmacology

081337991177

Microbiology

08123831710

Pharmacology
Obstetrics &
Gynecology
Microbiology

08179787972
081339550423

dr.Putu Ayu Asri Damayanti,M.Kes

Parasitology

085338565783

dr.Made Sudarmaja,M.Kes

Parasitology

08123953945

Faculty of Medicine UNUD,MEU

08179747502

Study Guide Infection and Infectious Diseases

~ FACILITATORS ~
REGULAR CLASS
NO
1
2
3
4
5
6
7
8
9
10
11
12

NAME
dr. Made Widhi Asih, Sp.Rad
dr. Muliani , M Biomed
dr. Ni Gusti Ayu Putri Mayuni, S.Ked
dr. Nyoman Suryawati , M.Kes, Sp.KK
dr. Ni Luh Ariwati
dr. Ni Luh Putu Ratih Vibriyanti Karna,
Sp.KK
dr. Ni Made Adi Tarini, Sp.MK
dr. Ni Made Dewi Dian Sukmawati,
Sp.PD
dr. I G.A. Indah Ardani, Sp.KJ
dr. Ni Nengah Dwi Fatmawati ,
Sp.MK, Ph.D
dr. Ni Nyoman Margiani, Sp.Rad
dr. Ni Putu Sriwidyani , Sp.PA

ENGLISH CLASS
NO
NAME
1
2
3
4
5
6
7
8
9
10
11
12

dr. I Gusti Made Gde Surya Chandra


Trapika , M.Sc
dr. Anak Agung Wiradewi Lestari , Sp
PK
dr. Nyoman Paramita Ayu, Sp.PD
dr. Ni Kadek Mulyantari , Sp PK
dr Wayan Citra Wulan Sucipta Putri
dr. A.A.Ngurah Subawa , Msi
Dr. dr. Bagus Komang Satriyasa,
M.Repro
dr. Pontisomaya Parami, Sp.An
dr. Pratihiwi Primadharsini, M.Biomol,
Sp.PD
dr. Putri Ariani, Sp.KJ
dr. I Gusti Ayu Widianti , M.Biomed
dr. Ni Putu Sri widyani , Sp.PA

Faculty of Medicine UNUD,MEU

GROUP
1
2
3
4
5
6
7
8
9
10
11
12

GROUP
1
2
3
4
5
6
7
8
9
10
11
12

DEPT

PHONE

Radiology

081916442626

Anatomy

08123930767

Andrology

081933113003

Dermatology

0817447279

Parasitology

08123662311

Dermatology

081337808844

Microbiology

081338675344

Interna

081805656501

Psychiatry

0361 8810404

Microbiology

087862200814

Radiology

081337401240

Anatomy
Pathology

081337115012

DEPT

PHONE

Pharmacology

081337991177

Clinical
Pathology

08155237937

Interna

08123837372

Clinical
Pathology

08123647413

Public Health

082140517310

Clinical
Pathology

08155735034

Pharmacology

081237166686

Anasthesi

08123661312/
08113800107

Interna

081805530196

Psychiatry

08123806397

Anatomy

08123921765

Anatomy
Pathology

085338565783

VENUE
3nd floor:
R.3.01
3nd floor:
R.3.02
3nd floor:
R.3.03
3nd floor:
R.3.04
3nd floor:
R.3.05
3nd floor:
R.3.06
3nd floor:
R.3.07
3nd floor:
R.3.08
3nd floor:
R.3.20
3nd floor:
R.3.21
3nd floor:
R.3.22
3nd floor:
R.3.23

VENUE
3nd floor:
R.3.01
3nd floor:
R.3.02
3nd floor:
R.3.03
3nd floor:
R.3.04
3nd floor:
R.3.05
3nd floor:
R.3.06
3nd floor:
R.3.07
3nd floor:
R.3.08
3nd floor:
R.3.20
3nd floor:
R.3.21
3nd floor:
R.3.22
3nd floor:
R.3.23

Study Guide Infection and Infectious Diseases

TIME-TABLE (Block Infection and infectious Diss.)


DAY/ DATE

Topic

Time
Regular
Class

English
Class

08.00-08.30

09.00-09.30

08.30-09.00

09.30-10.00

09.00-10.30

12.00-13.30

10.30-12.00

13.30-15.00

12.30-14.00

10.00-11.30

14.00-15.00

15.00-16.00

08.00-09.00

09.00-10.00

09.00-10.30

12.00-13.30

10.30-12.00

13.30-15.00

12.30-14.00

10.00-11.30

14.00-15.00

15.00-16.00

08.00-08.30

09.00-093.0

Thursday
Nov. 13th14

08.30-09.00

09.30-10.00

09.00-10.30

12.00-13.30

10.30-12.00

13.30-15.00

12.30-14.00

10.00-11.30

14.00-15.00

15.00-16.00

08.00-08.30

09.00-093.0

1
Tuesday
Nov. 11th 14

2
Wednesday
Nov. 12th 14

Faculty of Medicine UNUD,MEU

Lecture 1
Introduction to the
block (Agent ,Host
Environment, and
infection
manifestation)
Lecture 2
bacterial
classification

Learning
situation

Introduction to
the Block

Disc. Room

Class room

Lecture

Class room

Lecture
Lecture
Individual
Learning
Small group
discussion
Student
Project
Plenary

Lecture 6
Manifestation of virus

Plenary
Session

Individual
learning
Small Group
Discussion
Student
Project
Plenary
Lecture 4
Viral classification
Lecture 5
Mechanism of Viral
Pathogenesis

Class room

PIC

Prof. Dr. dr. Tuti


Parwati Merati,
SpPD, KPTI

dr. K. Januarta,
M.Kes
Individual
learning
Small group
discussion
Student
Project

Lecture 3
Mechanism of
bacterial
Pathogenesis

Place

Lecture

Class
Room
Class
Room
Class
Room
Class room

Facilitator
Prof. Dr. dr. Tuti
Parwati Merati,
SpPD, KPTI dr. K.
Januarta, M.Kes
Prof. Dr. dr. Tuti
Parwati Merati,
SpPD, KPTI dr. K.
Januarta, M.Kes
dr. Agus
Hendrayana, M.Ked

dr. Agus
Hendrayana, M.Ked
dr. Agus
Hendrayana, M.Ked
Dr. dr. Sri
Budayanti, Sp.MK
Dr.dr. Sri Budayanti,
Sp.MK

Disc. Room

Class room

Class
Room

Dr. dr. Sri


Budayanti, Sp.MK
Dr. dr. Sri
Budayanti, Sp.MK

dr.Agus somia,
Sp.PD

10

Study Guide Infection and Infectious Diseases

DAY/ DATE

Topic

Time
Regular
Class

English
Class

08.30-09.00

09.30-10.00

09.00-10.30

12.00-13.30

10.30-12.00

13.30-15.00

12.30-14.00

10.00-11.30

14.00-15.00

15.00-16.00

08.00-09.00

09.00-10.00

Monday
Nov. 17th14

09.00-10.30

12.00-13.30

10.30-12.00

13.30-15.00

12.30-14.00

10.00-11.30

14.00-15.00

15.00-16.00

08.00-09.00

09.00-10.00

Friday
Nov. 14th14

6
Tuesday
Nov. 18th14

7
Wednesday
Nov. 19th14

and bacterial infection


Lecture 7
Basic concept of
Parasitic Infections

Lecture
Individual
Learning
Small group
discussion
Student
Project

Lecture 8
Treatment of Viral
Infection (PK/PD)

Lecture 9
Treatment of
Microbacterial
Infections I (Type of
antimicrobacterial)
(PK/PD)

09.00-10.30

12.00-13.30

10.30-12.00

13.30-15.00

12.30-14.00

10.00-11.30

14.00-15.00

15.00-16.00

08.00-08.30

09.00-10.00

Lecture 10
Treatment of
Microbacterial
Infections II
(Resistance, rational
treatment, and drug
combination)

08.30-09.00

12.00-13.30

Lecture 11
Antimicrobial
susceptibly

Faculty of Medicine UNUD,MEU

Learning
situation

Place

Class room

Prof. dr. D.P.


Widjana, DAP&E,
Sp.Par.K

Disc. Room

Plenary

Class room

Lecture

Class room

Individual
learning
Small group
discussion
Student
Project
Plenary
Session
Lecture

PIC

dr.Agus
somiaSp.PD
Prof. dr. D.P.
Widjana, DAP&E,
Sp.Par.K
dr.Agus
somiaSp.PD
Prof. dr. D.P.
Widjana, DAP&E,
Sp.Par.K
Prof. IGM Aman,
Sp.FK

Disc. Room

Class room

Facilitator
Prof. IGM Aman,
Sp.FK
Prof. IGM Aman,
Sp.FK

Class room
Dr.dr. B.K.
Satriyasa,M.Repro

Individual
learning
Small group
discussion
Student
Project
Plenary
Session
Lecture

Individual
learning

Disc. Room
Class room
Class room

Dr.dr. B.K.
Satriyasa,M.Repro
Dr.dr. B.K.
Satriyasa,M.Repro
dr. Made Jawi,
M.Kes

dr. Ni Made adi


Tarini, Sp.MK

11

Study Guide Infection and Infectious Diseases

DAY/ DATE

Topic

Time
Regular
Class

English
Class

09.00-10.30

13.30-15.00

10.30-12.00

10.00-11.30

12.30-14.00

15.00-16.00

08.00-08.30

09.00-093.0

08.30-09.00

09.30-10.00

09.00-10.30

12.00-13.30

10.30-12.00

13.30-15.00

12.30-14.00

10.00-11.30

14.00-15.00

15.00-16.00

08.00-08.30

09.00-093.0

Friday
Nov. 21th14

08.30-09.00

09.30-10.00

09.00-10.30

12.00-13.30

10.30-12.00

13.30-15.00

12.30-14.00

10.00-11.30

14.00-15.00

15.00-16.00

8
Thursday
Nov. 20th14

Faculty of Medicine UNUD,MEU

Lecture 12:
Respond Host against
parasitic and clinical
manifestation
Lecture 13
Treatment of
parasitic infection
(PK/PD)

Learning
situation

PIC

Small group
discussion
Student
Project

Disc. Room

Facilitator

Class room

Plenary
Session
Lecture

Class room

dr. Ni Made adi


Tarini, Sp.MK dr.
Made Jawi, M.Kes
Dr. Made Jawi,
M.Kes
dr. I Made Susila
Utama,Sp.PD

Lecture

Class room

dr. I Gusti Made


Surya Candra
Trapika, M.Sc

Disc. Room

Facilitator

Class room

dr. I Made Susila


Utama,Sp.PD
dr. I Gusti Made
Surya Candra
Trapika, M.Sc
dr. I Made Susila
Utama,Sp.PD
dr. I Gusti Made
Surya Candra
Trapika, M.Sc
Dr.dr. Dewa Made
Sukrama, M.Si,
Sp.MK
Prof Dr.dr. IB
Rai,Sp.P

Individual
learning
Small group
discussion
Student
Project

Lecture 14:
The Role of Immunity
to infection (Basic)
Lecture 15: Infection
of Mycobacterium
(TBC)

Place

Class room

Plenary
Session

Class room

Lecture

Class room

Lecture

Class room

Individual
learning
Small group
discussion
Student
Project

Plenary
Session

Disc. Room
Class room

Class room

Dr.dr. Dewa Made


Sukrama, M.Si,
Sp.MK
Prof Dr.dr. IB
Rai,Sp.P
Dr.dr. Dewa Made
Sukrama, M.Si,
Sp.MK
Prof Dr.dr. IB
Rai,Sp.P

12

Study Guide Infection and Infectious Diseases

DAY/ DATE

10

Topic

Time

Tuesday
Nov. 25th14

12

Place

PIC

Regular
Class

English
Class

08.00-08.30

09.00-093.0

Lecture 16: Infection


of Mycobacterium
(Leprosy)

Lecture

Class room

dr. Dharma putra,


Sp.KK

08.30-09.00

09.30-10.00

Lecture 17:
Antimycobacterial
Drugs ( anti TBC, Anti
lepra) (PD/PK)

Lecture

Class room

dr. IB Ngurah, M.For

09.00-10.30

12.00-13.30

10.30-12.00

13.30-15.00

Disc. Room

Facilitator

12.30-14.00

10.00-11.30

Class room

14.00-15.00

15.00-16.00

08.00-08.30

09.00-093.0

dr. Dharma putra,


Sp.KK
dr. IB Ngurah, M.For
dr. Dharma putra,
Sp.KK
dr. IB Ngurah, M.For
dr. N Dwi
Fatmawati, Sp.MK,
Ph.D

08.30-09.00

09.30-10.00

09.00-10.30

12.00-13.30

10.30-12.00

13.30-15.00

12.30-14.00

10.00-11.30

14.00-15.00

15.00-16.00

08.00-08.30

09.00-093.0

Monday
Nov. 24th14

11

Learning
situation

Wednesday

Faculty of Medicine UNUD,MEU

Individual
Learning
Small group
discussion
Student
Project
Plenary

Class room

Lecture 18:
Control of
microorganism
(infection control)
Lecture 19:
Immunization in child

Lecture

Class room

Lecture

Class room

Lecture 20 : infections
in upper respiratory
tract (faringits,
tonsillitis, laringits,
otitis, mastoditis,
rhinitis, sinusitis,
furunkelitis)

Lecture

Lecture 20
Antiseptic and
disinfectant

dr. Dwi Lingga,


sp.A/ dr. W.
Gustawan,Sp.A
Dr Lely

Small group
discussion
Student
Project

Disc. Room

Facilitator

Class room

Plenary

Class room

Lecture

Class room

dr. N Dwi
Fatmawati, Sp.MK,
Ph.D
dr. Dwi Lingga,
sp.A/ dr. W.
Gustawan,Sp.A
dr. N Dwi
Fatmawati, Sp.MK,
Ph.D
dr. Dwi Lingga,
sp.A/ dr. W.
Gustawan,Sp.A
Dr.dr.B.K.
Satriyasa,M.Repro

13

Study Guide Infection and Infectious Diseases

DAY/ DATE

Nov. 26th14

13

Topic

Time

PIC

English
Class

08.30-09.00

09.30-10.00

09.00-10.30

12.00-13.30

Individual
learning

10.30-12.00

13.30-15.00

Small group
discussion

Disc. Room

Facilitator

12.30-14.00

10.00-11.30

Student
Project

Class room

14.00-15.00

15.00-16.00

Plenary

Class room

08.00-09.00

09.00-10.00

Lecture

Class room

Dr.dr.B.K.
Satriyasa,M.repro
dr Agus
Somia,Sp.PD
Dr.dr.B.K.
Satriyasa,M.repro
dr Agus
Somia,Sp.PD
dr.Dewa Ayu A. Sri
Laksmi,M.Sc,
M.Kes
dr. Putu Astri
Damayanti,M.Kes

09.00-10.30

12.00-13.30

Individual
learning

10.30-12.00

13.30-15.00

Small group
discussion

12.30-14.00

10.00-11.30

Student
Project

14.00-15.00

15.00-16.00

Plenary

Lecture 21
Universal Precaution

Lecture 22
Protozoa Infection I
(Malaria, Amoebiasis,
Leismaniasis,Tripano
somiasis,Toxoplasmo
sis, Trichomoniasis)

12.00-13.30

Friday
Nov. 28th14

Place

Regular
Class

Thursday
Nov. 27th14

14

Learning
situation

08.00-08.30

09.00-093.0

08.30-09.00

09.30-10.00

Faculty of Medicine UNUD,MEU

Lecture 23
Protozoa Infection II
(Management of
protozoa Infections)
Lecture 24
Infection of
Enterobacter
(Thypoid, C.
botulinum)

Lecture

Class room

dr Agus
Somia,Sp.PD
-

dr.Dewa Ayu A. Sri


Laksmi,M.Sc,
M.Kes
dr. Putu Astri
Damayanti,M.Kes
dr.Dewa Ayu A. Sri
Laksmi,M.Sc,
M.Kes
dr. Putu Astri
Damayanti,M.Kes

Middle block meeting


Lecture

Class room

dr Yuli Gayatri,
Sp.PD

Lecture

Class room

dr Agus
Somia,Sp.PD

14

Study Guide Infection and Infectious Diseases

DAY/ DATE

Topic

Time

Learning
situation

Place

PIC

Regular
Class

English
Class

09.00-10.30

12.00-13.30

Individual
learning

10.30-12.00

13.30-15.00

Small group
discussion

Disc. Room

Facilitator

12.30-14.00

10.00-11.30

Student
Project

Class room

14.00-15.00

15.00-16.00

Plenary

Class room

dr Yuli Gayatri,
Sp.PD
dr Agus
Somia,Sp.PD
dr Yuli Gayatri,
Sp.PD
dr Agus
Somia,Sp.PD

15

08.00-08.30

09.00-093.0

Lecture 25
Sepsis and
Bacteremia

Lecture

Class room

dr. Made Susila


utama, Sp.PD

Monday
Des. 1st 14

08.30-09.00

09.30-10.00

Lecture 26
Cutaneous Viral
Infection (Varicella,
Zoster, Herpes)

Lecture

Class room

dr.IGA Sumedha
Pindha, Sp.KK/dr.
Elis Indira,Sp.KK

09.00-10.30

12.00-13.30

Individual
learning

10.30-12.00

13.30-15.00

Small group
discussion

Disc. Room

Facilitator

12.30-14.00

10.00-11.30

Student
Project

Class room

14.00-15.00

15.00-16.00

Plenary

Class room

08.00-08.30

09.00-093.0

Lecture 27
Retroviral Infection
(HIV)

Lecture

Class room

dr. Made Susila


utama, Sp.PD
dr.IGA Sumedha
Pindha, Sp.KK/dr.
Elis Indira,Sp.KK
dr. Made Susila
utama, Sp.PD
dr.IGA Sumedha
Pindha, Sp.KK/dr.
Elis Indira,Sp.KK
Prof. Dr. dr. Tuti
Parwati Merati,
SpPD, KPTI

08.30-09.00

09.30-10.00

Lecture 28
Influenza

Lecture

Class room

Prof. Dr. dr. Tuti


Parwati Merati,
SpPD, KPTI

09.00-10.30

12.00-13.30

10.30-12.00

13.30-15.00

Disc. Room

Facilitator

12.30-14.00

10.00-11.30

Class room

Prof. Dr. dr. Tuti


Parwati Merati,
SpPD, KPTI

16
Tuesday
Des. 2nd 14

Faculty of Medicine UNUD,MEU

Individual
learning
Small group
discussion
Student
Project

15

Study Guide Infection and Infectious Diseases

DAY/ DATE

17
Wednesday
Des. 3th14

18

Topic

Time
Regular
Class

English
Class

14.00-15.00

15.00-16.00

08.00-08.30

09.00-093.0

08.30-09.00

09.30-10.00

09.00-10.30

12.00-13.30

10.30-12.00

13.30-15.00

12.30-14.00

10.00-11.30

14.00-15.00

15.00-16.00

08.00-09.00

09.00-10.00

09.00-10.30

12.00-13.30

10.30-12.00

13.30-15.00

12.30-14.00

10.00-11.30

14.00-15.00

15.00-16.00

08.00-030.00

09.00-09.30

08.300-09.00

09.30-10.00

Thursday
Des. 4th14

19
Friday
Des. 5th14

Faculty of Medicine UNUD,MEU

Lecture 29
Infection in children
(DBD, Difteri, sepsis,
Campak)
Lecture 30
infections in upper
respiratory tract
(faringits, tonsillitis,
laringits, otitis,
mastoditis, rhinitis,
sinusitis, furunkelitis)

Learning
situation

Place

PIC

Plenary

Class room

Prof. Dr. dr. Tuti


Parwati Merati,
SpPD, KPTI

Lecture

Class room

dr. Dwi Lingga,


sp.A/ dr. W.
Gustawan,Sp.A

Lecture

Class room

dr. Lely, Sp.THT

Individual
learning
Small group
discussion
Student
Project

Lecture 31
Zoonosis Infection
(Rabies,
Leptospirosis)

Class room

Plenary

Class room

Lecture

Class room

Individual
learning
Small group
discussion
Student
Project

Lecture 32
Principles of Fungal
Infection (Morphology
of Fungal)
Lecture 33:
superficial fungal
Infections (Tinea,
Tinea versikolor,
kadidiasis
mukokutaneous)

Disc. Room
Facilitator
dr. Dwi Lingga,
sp.A/ dr. W.
Gustawan,Sp.A
dr. Lely, Sp.THT
dr. Dwi Lingga,
sp.A/ dr. W.
Gustawan,Sp.A
dr. Lely, Sp.THT
Prof.Dr. dr. Raka
Sudewi, Sp.S (K)
Dr. dr. Sri
Budayanti, Sp.MK

Disc. Room

Facilitator

Class room

Prof.Dr. dr. Raka


Sudewi, Sp.S (K)
Dr. dr. Sri
Budayanti, Sp.MK
Prof.Dr. dr. Raka
Sudewi, Sp.S (K)
Dr. dr. Sri
Budayanti, Sp.MK
dr. Luh Ariwati

Plenary

Class room

Lecture

Class room

Lecture

Class room

Prof. M. Swastika
Adiguna

16

Study Guide Infection and Infectious Diseases

DAY/ DATE

Topic

Time
Regular
Class

English
Class

09.00-10.30

12.00-13.30

10.30-12.00

13.30-15.00

12.30-14.00

10.00-11.30

14.00-15.00

15.00-16.00

20

08.00-08.30

09.00-09.30

Monday
Des. 8th14

08.30-09.00

09.30-10.00

09.00-10.30

12.00-13.30

10.30-12.00

13.30-15.00

12.30-14.00

10.00-11.30

14.00-15.00

15.00-16.00

21

08.00-08.30

09.00-09.30

Tuesday
Des. 9th14

08.30-09.00

09.30-10,00

09.00-10.30

12.00-13.30

10.30-12.00

13.30-15.00

12.30-14.00

10.00-11.30

14.00-15.00

15.00-16.00

08.00-08.30

09.00-09.30

08.30-09.00

09.30-10.00

22

Faculty of Medicine UNUD,MEU

Learning
situation

Individual
learning
Small group
discussion
Student
Project

Lecture 34
Deep Fungal Infection
Lecture 35
Treatment of Fungal
Infection (PD/PK)

Disc. Room
dr. Luh Ariwati
Prof. M. Swastika
Adiguna

Plenary

Class room

Lecture

Class room

Lecture

Class room

dr. Luh Ariwati


Prof. M. Swastika
Adiguna
Prof.Dr.dr Tuti
Parwati,Sp.PD
dr.I B.Ngurah,
M.For/ dr. I Gusti
Made Surya Candra
Trapika, M.Sc

Disc. Room

Facilitator

Class room

Prof.Dr.dr Tuti
Parwati,Sp.PD
dr.I B.Ngurah,
M.For/ dr. I Gusti
Made Surya Candra
Trapika, M.Sc
Prof.Dr.dr Tuti
Parwati,Sp.PD
dr.I B.Ngurah,
M.For/dr. wiwiek
Indrayani, M.Kes
Dr. dr. I Made
Sudarmaja, M.Kes
dr. Kadek
Swastika,M.Kes

Plenary

Class room

Lecture

Class room

Lecture

Class room

Individual
learning
Small group
discussion
Student
Project

Lecture 38
Filariasis
Lecture 39
Dengue Viral Infection

PIC

Class room

Individual
learning
Small group
discussion
Student
Project

Lecture 36
Helminthes Infection
Lecture 37
Infection of
Nematoda, Cestoda
and Trematoda

Place

Disc. Room

Facilitator

Class room

dr. I Made
Sudarmaja, M.Kes
dr. Kadek
Swastika,M.Kes

Plenary

Class room

Lecture

Class room

Lecture

Class room

dr. I Made
Sudarmaja,
M.Kes/Staff
dr. K. Agus Somia,
Sp.PD
dr. Made Susila
Utama, Sp,PD

17

Study Guide Infection and Infectious Diseases

DAY/ DATE

Wednesday
Des. 10th14

23
Thursday
Des. 11th14

24

Topic

Time

Place

PIC

Regular
Class

English
Class

09.00-10.30

12.00-13.30

10.30-12.00

13.30-15.00

12.30-14.00

10.00-11.30

14.00-15.00

15.00-16.00

08.00-09.00

09.00-10.00

09.00-10.30

12.00-13.30

10.30-12.00

13.30-15.00

12.30-14.00

10.00-11.30

14.00-15.00

15.00-16.00

08.00-09.00

09.00-10.00

09.00-10.30

12.00-13.30

Individual
learning

10.30-12.00

13.30-15.00

Small group
discussion

Disc. Room

Facilitator

12.30-14.00

10.00-11.30

Student
Project

Class room

dr. Hariyasa
Sanjaya,Sp.OG

14.00-15.00

15.00-16.00

Plenary
Session

Class room

dr. Hariyasa
Sanjaya,Sp.OG

08.00-09.00

09.00-10.00

Lecture

Class room

Dr. dr. A.A.G.P.


Wiraguna, Sp.KK
(K), FINSDV

09.00-10.30

12.00-13.30

Individual
learning

10.30-12.00

13.30-15.00

Small group
discussion

Disc. Room

Facilitaor

Friday
Des. 12th14

25

Learning
situation

Monday
Des. 15th14

Faculty of Medicine UNUD,MEU

Individual
learning
Small group
discussion
Student
Project

Lecture 40
Treatment of
Helminthes Infection
(PK/PD)

Lecture 41
Overview of Puerperal
Infection

Lecture 42
Overview of Sexually
Transmitted Infection

Disc. Room

Facilitator

Class room

dr. K. Agus Somia,


Sp.PD
dr. Made Susila
Utama, Sp,PD
dr. K. Agus Somia,
Sp.PD
dr. Made Susila
Utama, Sp,PD
Dr.dr.
B.K.Satriyasa,M.Rre
pro

Plenary

Class room

Lecture

Class room

Individual
learning
Small group
discussion
Student
Project

Class room

Plenary
Session

Class room

Lecture

Class room

Facilitator
Class room

Dr.dr.
B.K.Satriyasa,M.Rre
pro
Dr.dr.
B.K.Satriyasa,M.Rre
pro
dr. Hariyasa
Sanjaya,Sp.OG

18

Study Guide Infection and Infectious Diseases

DAY/ DATE

Learning
situation

Topic

Time

Place

PIC

Regular
Class

English
Class

12.30-14.00

10.00-11.30

Student
Project

Class room

dr. A.A.G.P.
Wiraguna, Sp.KK

14.00-15.00

15.00-16.00

Plenary
Session

Class room

dr. A.A.G.P.
Wiraguna, Sp.KK

26

08.00-15.00

09.00-16.00

Laboratory

Friday
Des. 19th14
27

Practice 1 :
Laboratory diagnosis
of Microbial infection

I.B. Nyoman Putra


Dwija, S.Si,
M.Biotech

08.00-15.00

09.00-16.00

Laboratory

Monday
Des. 22th14
28

Practice 2 :
Laboratory diagnosis
of Microbial infection

I.B. Nyoman Putra


Dwija, S.Si,
M.Biotech

08.00-15.00

09.00-16.00

Practice 3 :
Laboratory Diagnosis
of Clinical Pathology

Laboratory

Dr Nyoman
Mahartini SpPK

08.00-15.00

09.00-16.00

Practice 4 :
Laboratory
Examination of
parasitic infectious

dr. Kadek Swastika,


M.Kes

08.00-15.00

09.00-16.00

Practice 5 :
Laboratory
Examination of
parasitic infectious ()

dr. Kadek Swastika,


M.Kes

Tuesday
Des. 23th14
29
Wednesday
Des. 24th14
30
Monday
Des. 29th14
31
Tuesday
Des. 30th14
32
Wednesday
Des. 31th14

Silent Day

EXAMINATION

BLOCK TEAM

MEETING OF STUDENT REPRESENTATIVES


In the middle of block period, a meeting is designed among the student representatives of
every small group discussion, facilitators and source person of the block. The meeting

Faculty of Medicine UNUD,MEU

19

Study Guide Infection and Infectious Diseases


discuss about the ongoing teaching and learning process, quality of facilitator and lectures
as a feedback to improve the next process.

MEETING OF THE FACILITATORS


All facilitators are invited to discuss all block activities with block contributors 1 week after
meeting of student representatives.

ASSESSMENT METHOD
1. Assessment will be held on 25th day of the block period. The time provision is 100
minutes. The number of MCQ is 100 with passing point 70.
2. Assessment in this block consists of:
SGD
: 5%
Student Project (Paper)
: 10%
Final exam
: 85%

STUDENT PROJECT
TITLE
(Subject/topic: choose from competency list)
Name:
NIM:
Faculty of Medicine, Udayana University
2011
1.
2.
3.
4.
5.

Faculty of Medicine UNUD,MEU

Introduction (Pendahuluan)
Content (Isi sesuai dengan judul paper)
Summary (Ringkasan)
References (Daftar pustaka): VanCouver style
Pages: 6-10, Spasi: 1.5, Time New Roman:12

20

Study Guide Infection and Infectious Diseases

Student Project
No

Topic

Kompetensi

Staphylococcus bacteremia
1. Staphylococcus: microbiologis aspect
2. Clinical spectrum of staphylococcus
infection
3. How are staphylococcus infection
diagnosed
4. Complication of staph infection
5. Treatment and prevention of staph
infection
Sinusitis
1. etiopathogenesis of sinus infection
2. clinical symptoms and sign of sinus
infection
3. management of sinus infection
4. complication of sinus infection
Otitis Media
1. Otitis media acute: etiopathogenesis
2. Otitis media acute: management
3. Otitis media purulenta
4. Otitis media khronic suppurative
5. Complication of acute titis media
Mastoiditis
1. etiologi
2. pathogenesis
3. diagnosis
4. management
5. complication
Peritonsilar abses
1. etiopathogenesis
2. clinical manifestation
3. diagnosis
4. management

Rheumatic fever
1. etiologi
2. pathogenesis
3. diagnosis
4. management
5. complication
Rheumatic disease
1. etiopathogenesis
2. clinical manifestation
3. management
4. complication
Meningitis Purulenta
1. ethiopathogenesis
2. clinical manifestation
3. diagnosis
4. management
5. complication

Faculty of Medicine UNUD,MEU

21

Study Guide Infection and Infectious Diseases


9

Meningitis serosa
ethiopathogenesis
clinical manifestation
diagnosis
management
complication

10

Plaque (Pes)
Etiologi
Transmisi
Management
Complication
Actinomycosis
Diagnosis (microbiology)
Clinical manifestation
Management
Chromoblastomycosis
Diagnosis (microbiology)
Clinical manifestation
Management

Maduromycosis
Diagnosis (microbiology)
Clinical manifestation
Management
Fever
- Patogenesis of fever
- Metabolic respon of fever
- How to measure body temperature and
fever pattern
- Algorithm management of acute fever
illness
- Management of fever
CMV
- CMV: virology
- Clinical spectrum of CMV
- CMV in immunocompetent
- CMV infection in immunocompromized
- Management of CMV

11

12

13

14

15

16

17

18

Malaria
- etiopatogenesis of severe malaria
- clinical spectrum of severe malaria
- malaria cerebral
- clinical approach management of severe
malaria
- malaria in pregnant
Dengue infection
- How to know warning simptom and sign
- severe dengue
- management of severe dengue
- management
Typhoid fever
- typhoid toxic

Faculty of Medicine UNUD,MEU

3A

22

Study Guide Infection and Infectious Diseases


19

20

21

22

23

24

25

26

27

Typhoid fever: intestinal complication

HIV/AIDS
- stigma of HIV/AIDS
- VCT
- PICT
- CST (care support treatment)
- ARV
Influenza
- seasonal influenza
- swine influenza
- Avian influenza
- Management
- Prevention
Acute Gastroenteritis
- watery diarrhea:
- inflammatory diarrhea
- how to assement of severity of
dehydration
- how to do rehydration
- how to do rectal swab
Yaws (patek)
- etiopatogenesis
- clinical picture
- laboratory confirmation
- Management
- Prevention
Rabies
- etiopatogenesis of rabies
- clinical picture of rabies
- laboratory confirmation of rabies
- how to manage dog bite
- how to giving vaccination (IM and
subcutans)
Candidiasis
- clinical spectrum of candida infection
- Laboratory confirmation
- Management
Leptospirosis
- etiopatogenesis
- clinical picture
- laboratory confirmation
- Management
- Prevention
Emerging and reemerging disease: legionalle
Clinical manifestation
Diagnosis microbiology
Management

3A

3B

Emerging and reemerging disease: Enterovirus


71 (HFMD)
Clinical manifestation
Diagnosis microbiology
Management

Faculty of Medicine UNUD,MEU

23

Study Guide Infection and Infectious Diseases


28

29

30

31

32

Emerging and reemerging disease: Coronavirus


(SARS)
Clinical manifestation
Diagnosis microbiology
Management
Emerging and reemerging disease: Bunyaviruses
(Hantavirus)
Clinical manifestation
Diagnosis microbiology
Management
Infeksi nosokomial
Definition
Manifestation
Management
Prevention
Antibiotic resisten
Mechanism of resistence
Rationale of using antibiotica
Prevention
How to using prudent antibiotic
Profile of antibiotic

Faculty of Medicine UNUD,MEU

24

Study Guide Infection and Infectious Diseases

LEARNING PROGRAM
LECTURE 1
Introduction to the block (Agent ,Host Environment, and infection
manifestation)
Oleh:
Prof. Dr. dr. Tuti Parwati Merati, SpPD, KPTI
================================================
Lecture 2:
Bacterial classification
Oleh:
dr. K.Januartha P. Pinatih, Mkes
1.
2.
3.
4.
5.

Describe relationship between microbes and human in health and disease


Explain normal human flora and opportunistic infections
Describe the establishment of microbial infection
Explain the difference between Gram-positive and Gram-negative bacterial cell wall !
Classify the spherical bacteria (cocci) into Gram-positive and negative group. List
their virulence factors and related diseases caused by them !
6. Classify the rod bacteria (bacilli) into Gram-positive and negative group. List their
virulence factors and related diseases caused by them !
7. List the important enteric bacteria (Enterobacteriaceae), their virulence factors and
related diseases !
8. Classify the anaerobic bacteria according to their capabilities to form spores. List
their virulence factors and related diseases caused by them !
9. Explain the spesific characteristic of Mycobacteria cell-wall and the implication to
their natural resistance !
10. Explain the virulence factors and pathogenesis of infection caused by Mycobacteria !

Lecture 3:
PATHOGENESIS OF BACTERIAL INFECTION
Made Agus Hendrayana
ABSTRACT
The pathogenesis of bacterial infection includes initiation of the infectious process
and the mechanisms that lead to the development of signs and symptoms of disease.
Characteristics of bacteria that are pathogens include transmissibility, adherence to host
cells, invasion of host cells and tissues, toxigenicity, and ability to evade the host's immune
system. Many infections caused by bacteria that are commonly considered to be pathogens
are inapparent or asymptomatic. Disease occurs if the bacteria or immunologic reactions to
their presence cause sufficient harm to the person.
Bacteria (and other microorganisms) adapt to the environment, including animals
and humans, where they normally reside and subsist. In doing so, the bacteria ensure their

Faculty of Medicine UNUD,MEU

25

Study Guide Infection and Infectious Diseases


survival and enhance the possibility of transmission. By producing asymptomatic infection or
mild disease, rather than death of the host, microorganisms that normally live in people
enhance the possibility of transmission from one person to another.
Some bacteria that commonly cause disease in humans exist primarily in animals
and incidentally infect humans. Other bacteria produce infection of humans that is
inadvertent, a mistake in the normal life cycle of the organism; the organisms have not
adapted to humans, and the disease they produce may be severe.
The clinical manifestations of diseases (eg, diarrhea, cough, genital discharge)
produced by microorganisms often promote transmission of the agents.
Many bacteria are transmitted from one person to another on hands. A person with S
aureus carriage in the anterior nares may rub his nose, pick up the staphylococci on the
hands, and spread the bacteria to other parts of the body or to another person, where
infection results. Many opportunistic pathogens that cause nosocomial infections are
transmitted from one patient to another on the hands of hospital personnel.
The most frequent portals of entry of pathogenic bacteria into the body are the sites
where mucous membranes meet with the skin: respiratory (upper and lower airways),
gastrointestinal (primarily mouth), genital, and urinary tracts. Abnormal areas of mucous
membranes and skin (eg, cuts, burns, and other injuries) are also frequent sites of entry.
Normal skin and mucous membranes provide the primary defense against infection. To
cause disease, pathogens must overcome these barriers.
Once in the body, bacteria must attach or adhere to host cells, usually epithelial cells. After
the bacteria have established a primary site of infection, they multiply and spread directly
through tissues or via the lymphatic system to the bloodstream. This infection (bacteremia)
can be transient or persistent. Bacteremia allows bacteria to spread widely in the body and
permits them to reach tissues particularly suitable for their multiplication and cause the
diseases.
Learning Task
Case :
A 35 years old female, a secretary at private company come to general practician
complained that she has unreasonable pain when urinate since 5 days. She feels pain too
at lower abdominal. The urine color is dark yellow and little bit cloudy. Other physical
examination results are normal. The practician ask for laboratory examination for urine
analysis and urine culture. After few days, the urine analysis shown that she has urinary
tract infection. The urine culture shown colonies of Escherichia coli bacteria and significant
as agent of infection.
Questions :
1. In this case, Escherichia coli as a pathogen bacteria. When is Escherichia coli called
as colonization bacteria?
2. Explain the differentiation between true pathogen and opportunistic pathogen!
3. Explain the pathogenesis how Escherichia coli can infect the urinary tract (from
transmission until infection and cause the disease) !
4. What are Escherichia colis virulence factors that can cause urinary tract infection?
5. Explain the microbial virulence factors that you know!
6. Explain the differentiation between exotoxins and endotoxin !
7. Describe how several pathogens are able to survive inside the macrophages !
8. Explain the routes of transmission that you know and give examples of each !

Faculty of Medicine UNUD,MEU

26

Study Guide Infection and Infectious Diseases


Self Assessment
1. Explain the meaning of this term above :
A.
B.
C.
D.
E.
F.
G.
H.
I.
J.
K.

Contamination
Colonization
Invasion
Infection
Pathogen
Carrier
Nonpathogenic
Opportunistic pathogen:
Pathogenicity:
Toxigenicity:
Virulence:

L. Symbiosis
M. Commensalism
N. Parasitism
O. Zoonoses
2. Give examples of attachment mechanism !
Reff :
Jawetz, Melnick, Adelberg. 2010. Chapter 9. Pathogenesis of Bacterial Infection in Medical
Microbiology, 25th Edition by Vishal . The McGraw-Hill Companies. Lange Microbiology.

Lecture 4
Viral classification
Oleh:
dr. Sri Budayanti, Sp.MK
=====================================================
Lecture 5
Mechanism of Viral Pathogenesis
Oleh:
dr. Sri Budayanti, Sp.MK

======================================
Lecture 6
Manifestation of virus and bacterial infection
dr.Agus somia, Sp.PD
======================================================
Faculty of Medicine UNUD,MEU

27

Study Guide Infection and Infectious Diseases

Lecture 7
Basic concept of Parasitic Infections
Oleh:
Prof. dr. D.P. Widjana, DAP&E, Sp.Par.K
===============================================================

Lecture 8
Treatment of Viral Infection (PK/PD)
Prof. dr. IGM Aman, Sp.FK
Most of antiviral agents exerts their actions on viral replication, at the stage of nucleic acid
synthesis ot the stage of late protein synthesis and processing. Most of antiviral agents
active against herpes viruses and against the Human Immunodeficiency Virus (HIV) are
antimetabolites, so that it must first undergo conversion to active forms, usually triphosphate
derivatives. One of the most important recent trends in viral chemotherapy has been
combination therapy, where treatment with combination result in greater effectiveness and
prevent or delay the emergence of resistance, especially in the treatment of HIV disease.
Such combination usually include two Nucleoside Reverse Transcriptase Inhibitor (NRTIs)
plus Protease inhibitor. In some combination regimens, a non nucleoside reverse
transcriptase inhibitor (NNRTI) has been used place of Protease inhibitor. Highly active
antiretroviral therapy (HAART) is recommended for AIDS patients.

Learning Task
A male patient, 30 year old, is HIV-positive, has a CD4 count 300/ul and a viral RNA load
500 copies/ml. The physician give him antiviral drug. Two weeks later he complained
anorexia, nausea, vomiting, and abdominal pain. His abdomen was tender in the epigastric
area. Finally the physician diagnose him as acute pancreatitis.
1. List drugs that have cross resistance with acyclovir, and explain the reason why
cross resistance happened? (Katzung p.824)
2. List and describe the drugs preserved for acyclovir resistant strain. (Katzung p.824)
3. In the treatment of HIV disease, the combination of antiviral is needed. Explain the
adventages of drug combination. In the case whats likely antiviral drug given by the
doctor.
4. How do you manage this patient?
Self assessment:
1. A patient suffering from herpes simplex, treated with acyclovir. But HSV is resistant
to acyclovir. The alternative drug can choose:
1. Ganciclovir
2. Valaciclovir
3. Famciclovir
4. Cidofovir

Faculty of Medicine UNUD,MEU

28

Study Guide Infection and Infectious Diseases


2. As antiviral, the clinical use of acyclovir are as follow:
1. Varicella
2. Retinitis by CMV (cytomegalovirus)
3. Herpes zoster
4. Reccurent herpes labialis
3. The antiviral that are good for treating hepatitis patient are:
1. Lamivudin
2. Ribavirin
3. Interferon
4. Stavudin
4. For treated AIDS patient a combination of antiviral are needed. The combination that
are effective for this patient are:
1. Indinavir + Didanosine + Lamivudin
2. Acyclovir + Amantadine + Zidovudine
3. Zidovudine + Didanosine + Nevirapine
4. Ganciclovir + Sorivudine + Cidofovir

Lecture 9
Treatment of Microbacterial Infections I (Type of antimicrobacterial)
(PK/PD)
Oleh:
Dr.dr. B.K. Satriyasa,M.Repro
Abstract
Many of microorganism are classified as either Gram-positive or Gram-negative. Both of
them could be differentiated by several respect, not least in the structure of the cell wall,
which has implications for the action of antibiotics. The cell wall of Gram-positive organisms
is a relatively simple structure and it consist of 50% peptidoglycan. The cell wall of Gramnegative organisms is much complex, so more difficult in penetrating by some antibiotics.
Antibiotic for which penetration is a problem include benzylpenicillin, methicillin, macrolides,
vancomycin, bacitracin, and novobiocin. There are many mechanisms of action of
antibiotics or antimicrobial drugs in killing or inhibited the bacterial growth such as: inhibit
cell wall synthesis, inhibit protein synthesis, as a antimetabolites, and inhibit microbial
nucleic acid metabolism. The emergence of microbial resistance pose a constant challenge
to the use of antimicrobial drugs. Mechanism of underlying microbial resistance to the cell
wall synthesis inhibitors include the production of antibiotic-inactivating enzymes, change in
the structure of target receptors, increased efflux via drugs transporters, and decreases in
the permeability of microbes cellular membranes to antibiotics. Strategies designed to
combat microbial resistance include the use of adjunctive agents that can protect against
antibiotic inactivation, the use of antibiotic combination and avoid the misuse of antibiotic.
Learning Task
A-36-year old woman recently treated for leukemia is admitted to the hospital with malaise,
chills, and high fever. Bram stain of blood reveals the presence of Gram negative bacilli.
The initial diagnosis is bacteremia. The records of the patient reveal that she had a severe
urticarial rash after oral penicillin V.
a. If you a medical doctor what antibiotic would you choose for this woman?
b. Explain the mechanism of action and adverse reaction of the drugs that you choosed

Faculty of Medicine UNUD,MEU

29

Study Guide Infection and Infectious Diseases


c. In your opinion is there appropriate if that pasien treated by Chloramphenicol? Explain
your answer.
Self assessment:
1. Which one of the following item is beta lactamase inhibitors:
a. Mafenide
b. Penicillin V
c. Clavulanic acid
d. Amoxycillin
e. Ofloxacin
2. Ciprofloxacin and the other fluoroquinolone mechanism of action is by:
a. Inhibiting the synthesis of bacterial protein
b. Inhibiting an enzyme deoxyribonucleic acid (DNA) gyrase
c. Interfering cell wall synthesis
d. Inhibiting the production of mycolic acid
e. Inhibiting enzyme dehydrofolate reductase
3. The following antibiotics inhibit bacterial protein synthesis and are considered as
bacteriostatic:
a. Azithromycin
a. Ofloxacin
b. Chlarithromycin
c. Ciprofloxacin
4. The following drugs are used for topical application:
a. Mafenide
b.
Sulfasalazine
c.
Silversulfadiazine
d.
Penicillin
5. Which ones are the contraindication of tetracycline:
a. Producing a yellow discoloration of teeth
b.
Growth retardation in relation to infant skeletal development
c.
Depression of bone growth
d.
Crystalluria
6. These statements are true about chloramphenicol:
a. It is a potent inhibitor of microbial protein synthesis
b. It binds reversibly to the p450 as sub unit of bacterial ribosomal
c. It inhibits the peptidyl transferase step of protein synthesis
d. It is a bacteriostatic broad spectrum antibiotic
7. Antibiotic that has ototoxic and nephrotoxic effect is:
a. Erythromycin
b. Streptomycin
c. Chloramphenicol
d. Amoxycillin
e. Clindamycin
Textbook
Source :
1. Katzung, B.G. 2001. Basic and Clinical Pharmacology. Eight Edition. Lange Medical
Books/McGraw Hill.

Faculty of Medicine UNUD,MEU

30

Study Guide Infection and Infectious Diseases


2. Katzung and Trevors. Pharmacology Examination and Board Review. Sixth
Edition.Lange Medical Books/McGraw-Hill.

Lecture 10
Treatment of Microbacterial Infections II (Resistance, rational
treatment, and drug combination)
Oleh: Dr. Made Jawi, M.Kes
=====================================================
Lecture 11
Antimicrobial susceptibly
Oleh: dr. Ni Made Adi Tarini, Sp.MK
======================================================
Lecture 12:
Respond Host against parasitic and clinical manifestation
dr. I Made Susila Utama,Sp.PD

===============================================
Lecture 13
Treatment of parasitic infection (PK/PD)
dr. I Gusti Made Surya Candra Trapika, M.Sc
Abstract
Malaria is the most important protozoal disease in tropical medicine. It is responsible
for 2 million deaths per year and much morbidity in the 200 million people worldwide who
are infected. Malaria is caused by four species of plasmodial parasites that are transmitted
by female anophelene mosquitoes. Anti malarial drugs are usually classified in terms of their
action against different stages of the parasite. They are used to prevent transmission or
cure malaria. The aim of prophylactic use is to prevent the occurrence of infection in a
previously healthy individual who is at potential exposure risk. Suppressive prophylaxis
involves the use of blood schizonticides to prevent acute attacks; causal prophylaxis
involves the use of tissue schizonticides or drugs against the sporozoite to prevent the
parasite established in the liver. Anti malarial drugs can be used curatively (therapeutically)
against an established infection. Suppressive treatment aims to control acute attacks,
usually with blood schizonticides; radical treatment aims to kill dormant liver forms, usually
with a hypnozonticide, to prevent relapsing malaria. Several classes of antimalarial drugs
such as chloroquine, amodiaquine, quinine, quinidine, mefloquine, primaquine, fansidar,
proguanil, artemisin, and atovaquone-proguanil. The effectiveness of anti malarial agents

Faculty of Medicine UNUD,MEU

31

Study Guide Infection and Infectious Diseases


varies between parasite species . In addition, drug resistance is an important therapeutics
problem, most notably with P falciparum.
Amoebic dysentery is caused by infection with Entamoeba histolytica, which is
ingested in a cystic form. Dysentery results from invasion of the parasite in the intestinal
wall. Occasionally, the organism insists in the liver, forming abscesses. E. Histolytica can
cause asymptomatic intestinal infection, mild to moderate colitis, severe intestinal infection,
ameboma, liver abscess and other extra intestinal infections. The choice of drugs for
amoebiasis depends on the clinical presentation. Drugs of choice for asymptomatic
intestinal infection are luminal agent such as diloxanide furoate, iodoquinol and
paromomycin; for mild to moderate intestinal infection are metronidazole plus luminal agent;
for severe intestinal infection and hepatic abscess are metronidazole plus luminal agent .
Toxoplasmosis is an infection caused by toxoplasma gondii parasite. Most people
have no symptoms because their immune system keeps the parasite from causing illness.
However, in people who have a weak immune system, toxoplasmosis can cause serious
medical problems, such as damage the eyes and brain. The immune system can become
weak for a number of reasons.The drug of choice for toxoplasmosis are pyrimethamine plus
clindamycin plus folinic acid
Learning Task
1. Ms. Dewi, a 25 year old student, presents with a four day history of high fever (40
C), general malaise , feeling intensely cold and shaking followed by profuse
sweating. He returned from Lombok island 3 weeks ago. She takes drugs for
malaria. Today she feel
dizziness, nausea, diarrhea, tinnitus, blurred vision ,
flushed, sweaty skin and impaired hearing.
Ouestions :
1. Which of the following antimalarial drugs causes a dose dependent toxicity ?
2. Describe the pharmacodynamic and pharmacokinetic properties of the major
antimalarial drugs (chloroquine, mefloquine, quinine, primaquine, and the
antifolate agents)!
The five star hotel usually has screening their food handler s every six months. Mr. Andi
had positive cysts amoebiasis without dysentery symptom.
Ouestions
1. Which of the following anti amoebiasis drugs can use to treat Mr. Andi ?
2. Describe the pharmacodynamic and pharmacokinetic properties of the major
amebicides (diloxanide, emetine, iodoquinol, and metronidazole) !
Mrs Ratna, a 28 years old, come to hospital policlinic with chief complaints had abortus for 3
times. She usually eat steak or satay and has many cat in her house. Doctor suspect she
had infected by toxoplasma gondii.
Ouestions
1. Identify the drugs useful for prophylaxis and treatment toxoplasmosis and know
their toxic effects !
Self assesment questions
1. Which of the following antimalarial drugs should be used for prophylaxis for travel to
the East of Lombok island ?
A. Chloroquine
B. Primaquine

Faculty of Medicine UNUD,MEU

32

Study Guide Infection and Infectious Diseases


C. Mefloquine
D. Hydroxychloroquine
E. Pyrimethamine
2. Which of the following drugs has a major side effect of hemolysis in persons with
G6PD deficiency?
A. Chloroquine
B. Primaquine
C. Mefloquine
D. Pyrimethamine
E. Doxcycline
3. Which of the following drugs is recommended as a single agent for oral treatment of
uncomplicated malaria due to chloroquine-resistent P falciparum strains ?
A. Doxycline
B. Iodoquinol
C. Primaquine
D. Proguanil
E. Quinine

4. Which of the following drigs is effective against E. histolytica and other protozoa that
live under anaerobic conditions?
A. Metronidazole
B. Pentamidine isethionate
C. Quinine
D. Eflornithine
E. Chloroquine
5. Which one of the following statements about amebicides is least accurate?
A. Diloxanide furoate is a luminal amebicide
B. Emetine is contraindicated in pregnancy and in patients with cardiac disease
C. Metronidazole has little activity in the gut lumen
D. Paromomycin is effective in extraintestinal amebiasis
E. Systemic use of iodoquinol may cause thyroid enlargement and peripheral
neuropathy

Faculty of Medicine UNUD,MEU

33

Study Guide Infection and Infectious Diseases

Textbook
Source :
3. Katzung, B.G. 2001. Basic and Clinical Pharmacology. Eight Edition. Lange Medical
Books/McGraw Hill.
4. Katzung and Trevors. Pharmacology Examination and Board Review. Sixth
Edition.Lange Medical Books/McGraw-Hill.

Lecture 14:
The Role of Immunity to infection (Basic)
Oleh:
Dr.dr. Dewa Made Sukrama, M.Si, Sp.MK

===============================================
Lecture 15:
Infection of Mycobacterium (TBC)
Prof Dr.dr. IB Rai,Sp.P
======================================================
Lecture 16:
Infection of Mycobacterium (Leprosy)
Dr. Dharma putra, Sp.KK
Morbus Hansen is an infectious disease primary affected the periphery nerve and
secondary affected skin and the other organ caused by Mycobacterium leprae. Readley and
Jopping classification is Tuberculoid-Tuberculoid (TT), BorderlineTuberculoid (BT),
Borderline-Borderline (BB), Borderlline-Lepromatous (BL), and Lepromatous-Lepromatous
(LL).
The 4 cardinal sign of Leprosy are: 1. Macula hypopigmented or erythematous skin,
2. Anaesthesi, 3. Enlargement of periphery nerve, 4. Acid Fast Bacilli (AFB) found from slit
skin smear. Diagnosis of leprosy is based on finding two from three cardinal sign of leprosy
or if only cardinal sign number 4.
There are two kind regimen therapy for leprosy i.e. the therapy for paucy bacillary
leprosy (TT, BT with AFB (-) are rimfapicin 600 mg a month and dafson6% (DDS) 100 mg a
day continuous for six month and for multi bacillary leprosy are rifampicin 600 mg a month,
clofacimin (lamprene) 300 mg a month continuous with lamprene 50 mg a day and dafson
(DDS) 100 mg a day continuous therapy for 12 months.

Faculty of Medicine UNUD,MEU

34

Study Guide Infection and Infectious Diseases


Kepustakaan
1. Andrews Diseases of the skin. Nine Ed
2. Leprosy. Third edition. Antony Bryceson
Intoroduction of Leprosy
1. Explain the etiology of leprosy ( My cobacterium leprae)
2. Explain the test for detection of M leprae : Zeihl-Neilsen
staining test,
histopathological examination, lepromin test, Gunawan test and anaesthetic test in
supporting the diagnosis of leprosy
3. Explain the classification of leprosy
4. Explain the clinical sign and symptom of leprosy
5. Explain the complications of leprosy
6. Explain the management of leprosy and the complications
Kasus
Seorang wanita, 35 tahun mengeluh ada bercak merah pada punggung kiri dan kanan
dengan batas tidak tegas, selain itu juga dijumpai bercak merah di wajah dan dada yg
tersebar simetris, kecil-kecil. Bercak merah tersebut tidak gatal. Selain itu dijumpai
penebalan pada cuping telinga kanan dan kiri serta alis mata rontok.
Pertanyaan :
a. Apa yang perlu ditanyakan lagi pada penderita tersebut?
b. Pemeriksaan apa saja yang diperlukan ?
c. Apa diferensial diagnosis Saudara ?
d. Apa diagnosa Saudara ?
Bagaimana penatalaksanaannya ?

Lecture 17:
Antimycobacterial Drugs ( anti TBC, Anti lepra) (PK/PD)
dr. IB Ngurah, M.For
The chemotherapy of infection caused by Mycobacterium tuberculosis is complicated
because: limited information about the mechanism of drugs action, the development of
resistance, the intracellular site of mycobacterial, the chronic mycobacterial disease and
many drug drug toxicities, and patient compliance. Chemotherapy of tuberculosis always
the use of drug combinations to delay of resistance and increased antituberculosis efficacy.
The 4 cardinal sign of Leprosy are: 1. Macula hypopigmented or erythematous skin, 2.
Anaesthesi, 3. Enlargement of periphery nerve, 4. Acid Fast Bacilli (AFB) found from slit
skin smear. Diagnosis of leprosy is based on finding two from three cardinal sign of leprosy
or if only cardinal sign number 4.
There are two kind regimen therapy for leprosy i.e. the therapy for paucy bacillary
leprosy (TT, BT with AFB (-) are rimfapicin 600 mg a month and dafson6% (DDS) 100 mg a
day continuous for six month and for multi bacillary leprosy are rifampicin 600 mg a month,
clofacimin (lamprene) 300 mg a month continuous with lamprene 50 mg a day and dafson
(DDS) 100 mg a day continuous therapy for 12 months.

Faculty of Medicine UNUD,MEU

35

Study Guide Infection and Infectious Diseases


Case 1:
A 40-year old man got cough since one month, lost of appetite and sweating every night.
After examination the physician diagnosed the patient as tuberculosis.
1. Describe the combination therapy for tuberculosis which used best
2. Explain the mechanism of action of each drugs
3. The therapy of tuberculosis need long time. Explain what is the reason.
4. Explain the interaction of isoniazid with phenytoin
5. Descrie the toxic effect of drugs for tuberculosis

6. List all drugs for leprosy


7. Describe the mechanism of action dapsose for leprosy
8. Describe why you use combination dapsone with rifampin and clofazimine
for leprosy
9. Describe the toxic effects of dapsone and treatment for erythema nodosum
10. Describe the pharmacological aspects of rifampin for leprosy
11. Describe the pharmacological aspects of clofazimine for leprosy
Self assessment:
1. Compare the fate of isoniazid in slow asetilator patient and rapid asetilator patient.
2. Isoniazid for tuberculosis is usully combined with vitamin B6. Describe the reason
3. Why do you choose pyrazinamide as primary drug for tuberculosis.

Lecture 18:
Control of microorganism (infection control)
dr. Ni Made Aditarini Sp. MK
ABSTRACT
Microorganism like viruses, bacteria, fungi and protozoans reproduce directly within
the host. They are usually small and have a short generation time. Recovery from infection
usually gives immunity against re-infection; in the case of viral infections this may be
lifelong. We know, the source of infection can be from community and hospital, while the
transmission of infection varies to depending from microorganism. The principle prevention
of infection must to know the kind of microorganism, transmission method and population of
infection. Among various major factors contributing to the emergence of infectious diseases,
the important ones are human demographics and behavior, industry and technology,
economic development and land use, globalization and international travel, microbial
adaptation and change, breakdown of public health measures, and economic disparity of
have and have-nots
One of the great achievements of applied medical research has been its success in
controlling so many infectious diseases; smallpox has been eradicated and other infections
are now controlled effectively in many parts of the world. This control has been
accomplished in three main ways by the use of chemotherapy, immunization and improving
the environment (e.g. better sanitation, nutrition)
In general, chemotherapy is used to control infectious diseases in individuals,
whereas immunization and environmental improvements are used for control in populations.
Understanding the ways in which these diseases arise, spread and can be controlled
requires detailed epidemiologic studies to provide an accurate basis for assessment of risks
and for planning intervention. These studies are based on knowledge of the infectious
agents and their patterns of association with their hosts, but require the collection and
analysis of data, in conjunction with the use of mathematical models, to produce useful
pictures of disease transmission and control. Where the causal links between a clinical

Faculty of Medicine UNUD,MEU

36

Study Guide Infection and Infectious Diseases


condition and an infectious agent or its mode of transmission are unknown, epidemiologic
investigations can establish this link and thus determine appropriate control strategies.
Learning Task
1. Describe risk factors are influence to community infection and hospital infection
2. Describe how infections flow through a host population .
3. Describe some strategies for control of infectious diseases.
4. Describe some factors are influence to spread of infection.
5. Describe some factors are influence the success of vaccination.
Self Assesment
1. Comparison of chemotherapy and vaccination
2. Explain the meaning of this term above :
a. Susceptible host
b. Incubation period
c. Latent period
d. Generation time
3. Mention some factors are important at vaccination gift
Reference :
Mims. Medical Microbiologi, 4th Edition, Mosby Elsevier. 2008. p.445-457 & p. 551-568

Lecture 19:
Immunization in child
dr. Dwi Lingga, sp.A/ dr. W. Gustawan,Sp.A
Abstract
Immunization is the process of artificially inducing immunity or providing protection from
disease. Active immunization is the process of stimulating the body to produce antibody and
other immune responses through administration of a vaccine or toxoid. Passive
immunization, the provision of temporary immunity by administration of preformed
antibodies derived from humans or animals. Biologic agents used to induce active
immunization include vaccines and toxoids. A vaccine is defined as a suspension of live
(usually attenuated) or inactivated microorganisms, or fractions there of, which is
administered to induce immunity and prevent infectious disease or its sequelae. There are
some diseases that can prevent with immunization. Polio, diphtheria, tetanus, pertusis,
tuberculosis, measles, hepatitis B, hepatitis A, influenza, meningitis caused by hemophilus
influenza type B. All vaccines may cause side effects, and immunization safety is a real
concern. Unlike most other medical interventions, vaccines are given to healthy people, and
people are far less willing to tolerate vaccines' adverse effects than adverse effects of other
treatments. As the success of immunization programs increases and the incidence of
disease decreases, public attention shifts away from the risks of disease to the risk of
vaccination, and it becomes challenging for health authorities to preserve public support for
vaccination programs.
Learning task
The baby, boy, 5 months old accompanied by his mother come to clinic to get immunization.
His mother told to doctor that her baby had fever after the first DPT immunization. Her baby
had fever until 380C. He has no seizure, no high crying but his mother worried about that
experience.
1. What is the explanation that you must tell to his mother?
2. How about the next immunization schedule?
3. What is contraindication for next immunization?

Faculty of Medicine UNUD,MEU

37

Study Guide Infection and Infectious Diseases

Lecture 20
Antiseptic and disinfectant
Dr.dr.B.K. Satriyasa,M.Repro
Abstract:
Disinfectants are chemical agent that inhibit or kill microorganism in an inanimate
environment. Antiseptics are disinfecting agent with sufficiently low toxicity for host cells that
they can be used directly on skin, mucous membranes or wound. Antiseptics and
disinfectants are extensively used in hospitals and other health care settings for a variety of
topical and hard-surface applications. A wide variety of active chemical agents (biocides)
are found in these products, many of which have been used for hundreds of years, including
alcohols, phenols, iodine, and chlorine.
A wide variety of active chemical agents (or biocides) are found in these products, many of
which have been used for hundreds of years for antisepsis, disinfection. Despite this, less is
known about the mode of action of these active agents than about antibiotics. In general,
biocides have a broader spectrum of activity than antibiotics, and, while antibiotics tend to
have specific intracellular targets, biocides may have multiple targets. The widespread use
of antiseptic and disinfectant products has prompted some speculation on the development
of microbial resistance, in particular cross-resistance to antibiotics. The process of
disinfectants prevent infection by reducing the number of potentially infective organism
either by killing, removing, or diluting them.
Antiseptics are disinfecting agents with sufficiency low toxicity for host cells that can used
directly in skin, mucous membrane, or wound. Disinfectants are strong chemical agents that
inhibit or kill microorganisms in an inanimate environment. Disinfectant and antiseptics do
not have selective toxicity, and their clinical use are therefore limited. Most antiseptics delay
wound healing. User of antiseptics and disinfectants need to consider their short-term and
long-term toxicity since they may general biocidal activity and may accumulate in the
environment or the body of the patients.
Learning Task
1. List the Disinfectants and antiseptics (Katzung & Trevors, Katzung, BG)
2. Explain the mechanism of action disinfectants and antiseptics (Katzung & Trevors,
Katzung, BG)
3. Describe the clinical use of disinfectants and antiseptics for nosocomial infection
4. Describe the side effect of disinfectants and antiseptics (Katzung & Trevors,
Katzung, BG)
Self assessment
1. Which one the following antiseptics promote wound healing?
A. Iodine
B. Alcohol
C. Hexachlorophene
D. Chlorhexidine
E. None of the above
2. . Which one the following antiseptics and disinfectant derivates of oxidizing Agent?
A. Iodine
B. Alcohol

Faculty of Medicine UNUD,MEU

38

Study Guide Infection and Infectious Diseases


C. Hexachlorophene
D. Chlorhexidine
E. Hydrogen peroxide
3. Alcohols are not used as sterilants because they are. EXCEPT:
A. They are sporicidal
B. Do not penetrate protein-containing organic material
C. May not be active against hydrophilic viruses
D. Lack residual action
E. They evaporate completely
4. Mechanism of action of povidone-iodine is to
A. Inhibitor of arabinosyl tranferase
B Inhibitor of thymidilate syntetase
C Inhibitor of protein kinase
D Denature of protein
E. Denature of lipid

Lecture 21
Universal Precaution
dr Agus Somia,Sp.PD
Learning task
Case 1
A 22-year-old male, work as an interns doctor in emergency care unit, had a patient with
suspected of HIV infection stage IV and Lung TB and chronic diarrhea. This doctor will do
the history-taking, physical examination and giving first aid to the patient
Learning Task:
1.
What is the type of exposure risk that may happen to this doctor?
2.
What is specific precaution that this doctor have to do to prevent cross
transmission?
3.
What are the kind of body protector that this doctor have to wear ?
4.
If this doctor have to take blood specimen with syringes needle to laboratory
examination, how to recapping needles in order to prevent the infection?
Self assessment:
1. Describe about:
a. Nosocomial infection
b. Kinds of nosocomial infection
c. How to do hygienic hand washing
d. How is the preparation and procedure of using sterile gloves?
e. How is the preparation and procedure unleashing sterile gloves?
2. Explain pathogenesis of:
a. Nosocomial blood stream infection

Faculty of Medicine UNUD,MEU

39

Study Guide Infection and Infectious Diseases

Lecture 22
Protozoa Infection I (Malaria, Amoebiasis,
Leismaniasis,Tripanosomiasis,Toxoplasmosis, Trichomoniasis)
Oleh:
dr.Dewa Ayu A. Sri Laksmi,M.Sc, M.Kes
dr. Putu Astri Damayanti,M.Kes
ABSTRACT
Protozoa are unicellular organisms that have trophozoite form with one or more nuclei
containing nucleoli or karyosome and bounded by a nuclear membrane and the usual
eukaryotic cytoplasmic organelles including mitochondria ribosomes and endoplasmic
reticulum. Trophozoite have a cell membrane but not cell wall. Most intestinal Protozoa also
develop cyst that are more resistant than the fragile trophozoite to drying, cold or other
environmental stresses.
Malaria and Toxoplasmosis well known as parasitic disease and have great impact
due to their worldwide distribution. Human malarial parasite were first seen in 1880 and
their development both in the anopheline mosquito and in the human blood stream was well
understood by 1900, however Several clinical syndromes known to be caused by infection
of malaria parasites were first recognized centuries before the discovery of their
pathogens.Consequently the diseases were referred to the type of febrile cycle. Quotidian,
tertian and quartan fevers denoted respectively 24-,48- and 72 hour cycles of fever. The
modern tendency is to refer the various types of malaria by the name of the agent.
Toxoplasma is caused by a coccidian parasite, Toxoplasma gondii. It has a
worldwide distribution and shows a broad host range from warm blooded animals to birds
and reptiles. Man acquires the infection indirectly by ingesting oocysts from contaminated
environments, by consuming Toxoplasma cysts from tissues of other intermediate hosts
such as cow, goat, chicken, duck, rabbit, by blood transfusion or transplantation, or
directly by transplacental infection
Human infection is generally asimptomatic and self limited except in
immunocompromised host, infection can disseminated and fatal. The prevalence of
antibody to toxoplasma in human and animal ranged from 2% to 75% in Southeast Asian
Countries. Cats are the definitive host of T. gondii; they are the only animals that pass
oocysts in their feces .
LEARNING TASKS PROTOZOA INFECTION
1. Case:
A 35 year old man present to primary public health service with one week history of
headache, fever, chills, sweats and myalgia. Patient history reveals that he just returned
from West Papua after 2 months lived there. He took chloroquine malarial prophylactic
irregularly. Physical examination showed raised body temperature (400C), a rapid pulse
rate, and generalized sweating. Complete Blood Count was ordered and demonstrated
intra erythrocyte organism.
a. Describe the laboratory examinations to define the diagnosis
b. When in blood smear demonstrate normal size erythrocyte containing crescent
shape gametocytes and multiple ring form within the blood cell,

What is the most likely diagnosed in this patient?

Faculty of Medicine UNUD,MEU

40

Study Guide Infection and Infectious Diseases

what is etiology of this cases and describe this parasite life cycle
c. Describe the pathogenesis of this disease

2.
3.
4.
5.

Please compare the morphology characteristic of 4 type of plasmodium in human.


Describe briefly about chagas disease.
Describe the life cycle of Trypanosoma cruzi and Leishmania donovani
Case
A previously healthy 28-year old man, who had recently returned from a trip to
Lombok, was seen by his family physician for crampy abdominal pain, malaise, slight
fever and bloody, mucoid diarrhea. Liquid stool specimens were collected and
submitted for culture for enteric bacterial pathogens as well as parasites. Stool cultures
were negative for bacterial pathogens, examination for ova and parasites was positive
for motile trophozoites in the saline wet amount, and ameboid trophozoites with finely
granular cytoplasm and ingested red blood cells in the permanent trichrome stain.
a. Describe the life cycle of parasites above !
b. Explain the pathogenesis of parasite above!
c. Describe infective stages of parasite above!

6. Describe the life cycle of Trichomonas vaginalis


7. Describe infective stages of Trichomonas vaginalis
8. Explain what the differences of Entamoeba histolytica and Giardia lamblia life cycle?
9. Describe the life cycle of Toxoplasma gondii
10. Explain transmission of Toxoplasma gondii infection
11. Explain why toxoplasma infection became latency?

Lecture 23
Protozoa Infection II (Management of protozoa Infections)
dr Yuli Gayatri, Sp.PD
Objectives

To describe name the 4 important members of the Genus Plasmodium


To known which from of Malaria is most dangerous and why.
To describe disease that Malaria most commonly mimic
To understand how Malaria is diagnosed
To describe the current recommendation for Malaria treatment and what factors
dictate the regimen of choice
Recognized when should chemoprofilaxis be begun and how long after completion
of a trip to an endemic area should preventive therapy be continued
To describe clinical presentation of Amoebiasis/ Toxoplasmosis/ Trichomoniasis
To know how Amoebiasis / Toxoplasmosis/ Trichomoniasis
Is diagnosed?
To describe what are the treatment of choice of Amoebiasis Toxoplasmosis/
Trichomoniasis

Faculty of Medicine UNUD,MEU

41

Study Guide Infection and Infectious Diseases


Case 1:
A 21-years-old man complained with fever for 6 days prior to admission, relapsing chills,
muscle eches and lost of appetite. He took several day trip to Lombok island. They noted
some mosquito bites and ate some fruits on the island. About 3 days into the illness He
became jaundice and began passing dark urine. They sought treatment from local Lombok
physician, who diagnosed hepatitis secondary to ingestion of toxic food. Two days later He
was referred to Sanglah Hospital for intensive treatment. Based on an initial examination,
patient was conscious, look pale and icteric, body temperature was 38,5 C.
Learning task:
1. Find key words related to this case
2. Describe condition related to key words
3. Define organ system that involved in this condition and find probably cause of
the key words
4. Define differential diagnosis and other examinations to support the diagnosis
5. Describe kinds of laboratory examination to diagnose malaria e.q. blood
smear, thick smear, rapid test, etc
6. Define management of this case
7. Define complication and prognosis
8. Define prevention based on individual, family, and community
Self assessment:
1. Describe kinds of plasmodium
2. Describe pathogenesis of malaria
3. Describe diagnosis of malaria
4. Describe pathogenesis of complication
5. Define management of uncomplicated malaria
6. Define management of malaria with severe complicatio

Lecture 24
Infection of Enterobacter (Thypoid, C. botulinum)
dr Agus Somia,Sp.PD
Case 1
A 22-year-old male, with feeling generally unwell with fever, headache, malaise and
diarrhea. the onset of fever since 7 days ago. His body temperature was 39 degree celcius,
blood pressure 120/80 mmHg, Pulse rate 100 beat per minute.
Learning Task:
5.
Define and describe others symptoms related to the patients that should be
asked to this patient
6.
Describe physical examination to support diagnosis of this patient.
7.
What is possibly diagnosis of this patient?
8.
Describe differential diagnosis of this case
9.
Describe laboratory and other examination to support the diagnosis
10.
Describe management of this patient
11.
Describe how to explain to this patient about prognosis of patient`s disease
Case 1:
A 42-year-old man complained with diarrhea since last night. His diarrhea was 10 times.
Diarrhea has accompanied with nausea, abdominal pain, and malaise. No history of fever
and stomachache. He is a salesman. He took medicine to retrieve his diarrhea, but it does
not work.

Faculty of Medicine UNUD,MEU

42

Study Guide Infection and Infectious Diseases


Learning task:
1. Define other sign and symptoms from this patient
2. Describe physical examination must be done to this patient
3. Describe laboratory examination and other examination must be done to support
diagnosis
4. Describe management of this patient
5. Describe plan of therapy based on priority on this patient
Self assessment:
3. Explain pathogenesis of:
a. Bacillare Dysentriae
b. Typhoid fever
c. Cholera
d. Clostridium difficile associated diarrhea
4. Describe and interpret cerebrospinal fluid (CSF) examination in bacterial meningitis,
viral meningitis, tuberculous meningitis, and streptococcal meningitis.
5. Define signs and symptoms of:
a. Typhoid fever
b. Bacillare dysentriae
c. Cholera
d. Clostridium difficile associated diarrhea
6. Differentiate clinical sign and symptoms of diarrhea caused by bacillare dysentriae,
cholera and Clostridium associated diarrhea
7. Describe steps for how we doing rehydration
8. Define management of these patients:
a. Typhoid fever
b. Bacillare dysentriae
c. Cholera
d. Clostridium difficile associated diarrhea
9. Describe about complication of
a. Typhoid fever
b. Bacillare dysentriae
c. Cholera
d. Clostridium difficile associated diarrhea

Lecture 25
Sepsis and Bacteremia
dr. Made Susila utama, Sp.PD

============================

Faculty of Medicine UNUD,MEU

43

Study Guide Infection and Infectious Diseases

Lecture 26
Cutaneous Viral Infection (Varicella, Zoster, Herpes)
Oleh:
dr.IGA Sumedha Pindha, Sp.KK/dr. Elis Indira,Sp.KK
Learning task
An adult woman, 45 years old came to clinic with chief complaints group of small blister in
right side of the back since 3 day ago then the lession spread to the right waist and right
chest. This complain is accompanied with burning sensation. One day before the blister
appeared patient had fever. History of the same disease was denied. History of taking
medicine before was denied.
1. What should we asked to the patient in the anamnesis ?
2. Describe the effloresensi in physical examination.
3. What are the differential diagnosis in this patient ?
4. What Laboratory tests is needed to confirm diagnosis of this case?
5. What is the diagnosis of this patient ?
6. Mention about complication of this disease
7. What is the prognosis of the disease ?
8. What is the treatment of this case?
9. What advice we can give to the patient ?
Self Assasement
1. What kind of diseased that can caused by the herpes virus group and what is the nature
virus of this group?
2. What is the majority characteristic of these group of virus?
3. What is clinical manifestation of varicella ?
4. How is pathogenesis of Herpes Zoster infection?
5. Mention about trigger factor the emergence of lesions in herpes simplex
6. What are the complication that occurs in Herpes Zoster?
7. Mention about complication that could occur happens when pregnant women suffer from
varicella
8. What is the management of skin diseases caused by viruses?

Lecture 27
Retroviral Infection (HIV)
Oleh:
Prof. Dr. dr. Tuti Parwati Merati, SpPD, KPTI

===============================================

Faculty of Medicine UNUD,MEU

44

Study Guide Infection and Infectious Diseases

Lecture 28
Influenza
Oleh:
Prof. Dr. dr. Tuti Parwati Merati, SpPD, KPTI
INFLUENZA
Abstract
Influenza virus infection, one of the most common infectious diseases, is a highly
contagious airborne disease that causes an acute febrile illness and results in variable
degrees of systemic symptoms, ranging from mild fatigue to respiratory failure and death.
These symptoms contribute to significant loss of workdays, human suffering, mortality, and
significant morbidity. Accurately diagnosing influenza A or B infection based solely on clinical
criteria is difficult because of the overlapping symptoms caused by the various viruses
associated with upper respiratory tract infection (URTI). In addition, several serious viruses,
including adenoviruses, enteroviruses, and paramyxoviruses, may initially cause influenza
like symptoms. The early presentation of mild or moderate cases of flavivirus infections (eg,
dengue) may initially mimic influenza. For example, some cases of West Nile fever acquired
in New York in 1999 were clinically misdiagnosed as influenza. Patients with influenza
frequently present with various symptoms shared by many other viral infections. In the
northern and southern hemispheres, these symptoms are more common in the winter
months.Influenza virus is a single-stranded RNA virus, divided into type A, B, and C where
structurally and biologically similar but vary antigenically. It is family of Orthomyxoviridae.
The most common prevailing influenza A subtypes that infect humans are H1N1 and H3N2.
Each year, the trivalent vaccine used worldwide contains A strains from H1N1 and H3N2,
along with an influenza B strain. Influenza virus infection occurs after transfer of respiratory
secretions from an infected individual to a person who is immunologically susceptible. If not
neutralized by secretory antibodies, the virus invades airway and respiratory tract cells.
Once within host cells, cellular dysfunction and degeneration occur, along with viral
replication and release of viral progeny. Systemic symptoms result from inflammatory
mediators, similar to other viruses. Influenza A is generally more pathogenic than influenza
B. Recently, mutation of influenza A virus cause the emergence of new strain of virus which
cause specific influenza such as Birds flue (H5N1) and Swine flue or Novel H1N1.
Case:
A man of 40 year-old came to hospital complaining fever, headache, sore throat and
myalgia since 4 days . He just come from Hong Kong about a week ago. He also had
cough, and feeling very weak.

Lecture29
Infection in children (DBD, Difteri, sepsis, Campak)
dr. Dwi Lingga, sp.A/ dr. W. Gustawan,Sp.A
Case 1

Faculty of Medicine UNUD,MEU

45

Study Guide Infection and Infectious Diseases


A boy, 6 years 6 months has come to our clinic with swelling and pain on the cheek under
right ear since 2 days ago. He has fever since 4 days ago. He hasnt coryza, cough and
cold. His appetite was decrease. His friends on the school have the same complaint.
1.
2.
3.
4.
5.

What is the close diagnosis of this case?


What is the differential diagnosis?
What is the laboratory support needed?
What is the therapy?
What are the complications of this case?

Case 2
A boy comes to my clinic with five days of fever as the chief complaint. He was currently 6
years and 10 months old and a first grade of elementary school student. Fever was
speaking immediately and has resolved one day before the doctor visited. This morning the
fever reappears, giving a pattern of saddle back fever, which is accompanied with
headache, muscular pain, articular and vertebra pain, retro orbital pain, nausea, vomiting
and skin rashes. The skin rash appeared at the beginning of the disease, but subsequently
vanished without any marks. In physical examination, the child looked compos mentis,
mildly ill with fever of 38.8 0C. Dermatological examination reveals skin rash, mainly on the
legs, foot soles and palms, the pharynx was slightly hyperemic and there is no palpable
enlarge lymph nodes on the neck. Auscultator finding of the heart and lungs were within
normal limits. Abdominal examination revealed epigastrial and right upper quadrant
tenderness on palpation. There was no liver enlargement. No significant finding existed on
the extremities, except for the positive tourniquets test.
1. What is the close diagnosis of this case?
2. What is the differential diagnosis?
3. What is the laboratory support needed?
4. What is the therapy?
5. What are the complications of this case?
Case 3
A girl, 2 years, comes to clinic with fever and rash as the chief complaint. Fever was appear
from 5 days ago and rash appear since yesterday, which is accompanied with headache,
cough, muscular pain, nausea, vomiting and red of her eyes. The rash phase is
accompanied by high fever. The macular rash begins on the head (above the hairline) and
spreads over of the body in 24 hours in a descending fashion.
1. What is the close diagnosis of this case?
2. What is the differential diagnosis?
3. What is the laboratory support needed?
4. What is the therapy?
5. What are the complications of this case?
Case 4
A boy, 3 years, comes to emergency department with unconsciousness since 2 hours ago.
This complaint suddenly occurs when his mother talk to him. He is no response to talk, no
move, and his eye look opened. He had fever since 5 days ago and still high until now. His
temperature was unstable, it was decrease after drink parasetamol, and increase again few
hour after that. The earliest symptoms are weakness, nausea or abdominal pain, and
headache.
1. What is the differential diagnosis?
2. What is the other data needed to complete this case?
3. What is the laboratory support needed?
4. What is the therapy?
5. What are the complications of this case?

Faculty of Medicine UNUD,MEU

46

Study Guide Infection and Infectious Diseases

Lecture 30
Zoonosis Infection (Rabies, Leptospirosis, Listeriosis)
Prof.Dr. dr. Raka Sudewi, Sp.S (K)
dr. Sri Budayanti, Sp.MK
Infections in central nervous system have certain unique characteristics. First, they
occur within an anatomic closed space. Secondly, the natural history of illnesses due to
CNS infection often differs strikingly from that of those due to infection at other sites, even
when caused by the same organism. Thirdly, many CNS infections cause high mortality the
patients survives, serious sequelae after resolution of the acute infections.
There are four cardinal manifestation of NS infection are: fever, headache, alteration
of mental status, and focal neurologic signs. Sometimes, these signs can be found in
noninfectious CNS syndromes. The time course of disease is especially important in the
evaluation of disease affecting the CNS. The date of onset, temporal relationship to
presdiposing factors, rate of progression, time to reach the peak of severity, time needed to
respond to treatment, and rate of resolution are all highly informative.
Infections of CNS can be caused by bacteria (pyrogenic infections), fungal,
spirochetal, parasitic, and sarcoid. Pyrogenic infections of CNS such as bacterial meningitis,
septic thromboplebitis, brain abscess, epidural abscess, and subdural empyema. The
granulomatous infections of CNS such as tuberculosis, syphilis, and other spirochetal
infections, and fungal infections.

Case 1
A 27 year-old man, Balinese, Hindu with unconciousness in emergency room Sanglah
Hospital. From his family told that it was convulsion at least for 3 hours before arrived in
hospital. From physical examination, axillary temperature 39,50C.
Learning Task:
12.
Define and describe others symptoms related to the patients that should be
asked to his family
13.
Describe physical examination to support diagnosis of this patient.
14.
What is possibly diagnosis of this patient?
15.
Describe differential diagnosis of this case
16.
Describe laboratory and other examination to support the diagnosis
17.
Describe management of this patient
18.
Describe how to explain to his family about prognosis of patient`s disease
Self assessment:
10. Describe about:
a. Meningitis
b. Encephalitis
c. Meningoencephalitis
d. Myelitis
e. Cerebral abscess
11. Explain pathogenesis of bacterial meningitis

Faculty of Medicine UNUD,MEU

47

Study Guide Infection and Infectious Diseases


12. Describe and interpret cerebrospinal fluid (CSF) examination in bacterial meningitis,
viral meningitis, tuberculous meningitis, and streptococcal meningitis.
13. Define signs and symptoms of:
a. Meningitis
b. Cerebral abscess
c. Acute Anterior Poliomyelitis
d. AIDS Dementia Complex
e. Cerebral cysticercosis
14. Differentiate clinical sign and symptoms of bacterial meningitis, viral meningitis,
tuberculous meningitis, and streptococcal meningitis
15. Define management of these patients:
a. Herpes Simplex Encephalitis
b. Tuberculous meningitis
c. Cerebral abscess
16. Define prognosis of:
a. Acute anterior poliomyelitis
b. Tuberculous meningitis
c. Cerebral abscess
17. Describe kinds or types of neurosyphilis
18. Describe about tabes dorsalis

Lecture 31
Principles of Fungal Infection (Morphology of Fungal)
dr. Luh Ariwati
Abstract :
Fungi are eukaryotic micro-organism, have a nucleus containing their DNA and a
RNA nucleolus, and cytoplasma. Surrounding them is plasmalemma which containing
ergosterol and out side plasmalemma is a rigid cell wall. Fungi do not contain chlorophyl
and cannot synthesize macro molecules from carbon dioxide and energy derived from light
rays, therefore all fungi lead a heterotrophic existence in nature as saprobes, commensals
or parasites.

Fungi can be divided into two basic morphologic form: yeast and hyphae.
Yeast are unicellular and reproduce asexually by budding and most fungi have branching,
threadlike tubular filaments called hyphae. Dimorphic fungi exist in both form. All fungi
reproduce by asexual processes and most can reproduce by sexual mechanism.
The fungi contribute to food spoilage, destroy textile, etc. As saprobe, they share
with bacteria in the decay of complex plant and animal remains in the soil. Fungi used in
production of antibiotics, products of fermentation such as beverages, soy sauce etc. Fungi
are free living and abundant in nature and a few live in normal flora of humans. Thousands
of species have been known, but less than 100 are cause diseases in humans. The effects
of fungi on humans are numerous such as mycotoxicosis, hypersensitivity and colonization
of fungi with resultant diseases.
Humans have good barriers against fungal infection such as intact skin, mucosal
surfaces, saliva, normal bacterial flora etc. Healthy, immunocompetent people have a high
innate resistant to fungi even though they are constantly exposed to the propagules of fungi.
Infections and diseases occur when there are disruptions in the protection barrier of skin

Faculty of Medicine UNUD,MEU

48

Study Guide Infection and Infectious Diseases


and mucus membrane or defect in immunity system. The characteristic of fungal pathogens
categorized into groups according to tissue that they colonize: superficial, cutaneous,
subcutaneous and systemic mycosis. Fungal infections that occur only because of
compromising situations are categorized as opportunistic mycosis.
Learning task:
1. Describe the structure of fungi
2. Explain terms used in medical mycology : yeast, hyphae, mycelium and dimorphic
fungi.
3. Describe the mechanisms of fungal pathogenesis
4. Describe the effect (medical importance) of fungi on humans
5. Describe the laboratory diagnosis of fungal diseases

Lecture 32:
superficial fungal Infections (Tinea, Tinea versikolor, kadidiasis
mukokutaneous)
Prof. M. Swastika Adiguna
Male 35 years old came to Dermatology Clinic with chief complain itching in the sites of
neck, upper, lower extremitas, trunk, and inner surfaces of the thigh especially during the
hot climate. It began as a small erythematous and scaling or vesicular and crusted patch
that spreads peripherally and partly clear in the centre. These lesion may be slightly
elevated particularly at the border, where they more inflamed and scaly.
1.
2.
3.
4.
5.
6.

Please explore another history to complete anamnesis


What kind of clinical examination will you do?
What kind of laboratory examination will you do.
What was the diagnosis of this patient?
Describe your planning therapy for this patient.
Describe your planning education for this patient.

Self assessment:
1. What in the definition of dermatophytosis (tinea or ring worm)
2. Please, describe the fungi of dermatophytes
3. what is the differential diagnosis of dermatophytosis
Please describe the antifungal therapy

Lecture 33
Deep Fungal Infection
Oleh:
Prof.Dr.dr Tuti Parwati,Sp.PD
Abstract
Fungal infections have become increasingly frequent especially in immune compromised
host such as AIDS, cancer patients, organ transplantation , and also as a consequent of
the availability of advanced medical technology which allow to do more invasive
treatment using more invasive instruments. Systemic fungal infections (SFI) or invasive
fungal infection are a significant cause of morbidity and mortality among immune
compromised patients, such as HIV-infected individuals, cancer patients, neonates and
patients in the intensive care unit.

Faculty of Medicine UNUD,MEU

49

Study Guide Infection and Infectious Diseases


The infections considered as nosocomial infections in the hospital for patients who have risk
factors such as immune compromised patients. The aetiology are : Predominant fungi :
Candida (C): such as C. Albicans, C. glabrata, C. tropicalis and C.parapsilopsis,
Aspergillus spp.and Cryptococcus spp., Emerging fungi : Fusarium spp., and Rhizopus
spp. and Endemic fungi : Histoplasma capsulatum, Blastomyces dermatitidis and
Coccidioides immitis
Clinical Diagnosis: There is no specific sign and symptoms of systemic fungal infection.
That is why suspected clinical diagnosis of systemic fungal infection is frequently late. Its
resemble bacterial infections, such as severe sepsis, septic shock and multi organ failure.
Alertness to this infection will comes late though sign and symptoms appear early. In many
cases the diagnosis was done per exlusionem. Diagnosis should be considered in patient
with risk factors has the signs of systemic infection despite adequate antibiotics.
Risk Factors : Patients at the Intensive Care Unit (ICU), Colonization of skin and mucous
membranes with Candida, Alteration of natural host barriers (wounds, surgery), The
colonized Candida might enter the blood stream when microbes balance disturbed by
antibiotics and the barrier altered, eg. Indwelling central/peripheral catheter, Neutropenia
(hematology disorders), Malignancy, Post chemo/radiation therapy, Organ Transplant
patients, HIV infection.
Portal entry : Gastro Intestinal tract, skin, urogenital tract, Catheter related
Case study:
Female 65 year-old consulted from a private hospital with chief complain : fever since 5
days ago. Other complain: coughing -7 days, has been treated with AB injection, improved ,
but worsening in the last 2 days. History of chronic diarrhoea, but not now. Loss of appetite
and Slight odinophagia. Physical examination : decreased of consciousness, high fever
(39,8 C), BP: 100/60 mm Hg, Pulse 108/min, RR 24/min. Heart : WNL, Ronchi +/+
Learning Task
What is DD of this patients?
Chest X ray : infiltrates on both middle and lower lung, increased hilar marking and
emphysematous lung. Lab. Test : CBC : Hb 7,2 g/dl, PLT 140, TLC 0,7 x 103 /ml, WBC 4,5
x 1000/ml, Other physical examination revealed : oral mucosa !! , tounge : coated.
What lab test do you suggest ?

Faculty of Medicine UNUD,MEU

50

Study Guide Infection and Infectious Diseases

Lecture 34
Treatment of Fungal Infection (PK/PD)
dr.I B.Ngurah, M.For/ dr. I Gusti Made Surya Candra Trapika, M.Sc
Abstract
Chemotherapy that are used as antifungal agents difficult to treat fungal infection
particularly in the immunocompromised or neutropenic patient. Drugs for systemic fungal
infections are amphotericin B, fluocytosine, and azole antifungal agents. Systemic drugs for
superficial fungal infections are griseofulvin, terbinafine, and azoles. Topical drugs for
superficial fungal infections are nystatin, miconazole, clotrimazole, haloprogin, tolnafnate,
and undecylenic acid. Only few drugs are available for tretament of systemic fungal
infections. Ergosterol is a sterol that is unique to the fungal cell membrane. The
predominant sterol of human cells is cholesterol.
A 25-year old woman with she feel itchy since 2 weeks ago and changes brown nail colour.
The physician diagnosed as dermatophytes of the nail.
1. What kind of antifungal the best to be used.
2. How is the mechanism of action of the drug?
3. Describe the pharmacokinetic of the drug
4. What will be happened if the drug is given concomitanly with coumarin?
Self assessment:
1. List the systemic antifungals for systemic fungal infections and superficial fungal
infections
2. List the topical drugs for superficial fungal infections
3. Explain the mechanism of action systemic antifungals and superficial antifungals
4. Explain the pharmacokinetics of amphotericin B, fluocytosine, fluconazole,
itraconazole, ketokonazole, griseofulvin, and terbinafine.
5. describe the clinical uses of systemic antifungals and superficialis antifungals.
6. Describe the toxic effects of systemic antifungal and superficial antifungals.

Lecture 35
Helminthes Infection
dr. I Made Sudarmaja, M.Kes
In the National Standard of Competency of Undergraduate Medical Education, the core
content of curriculum related to helminthic infections has been identified. This core must be
well mastered by the students. Although hundred or more species are identified as a
helminthes of medical importance, however, only a few of those are considered as core.
The core species are: (1) Ascaris lumbricoides (2) Trichiuris trichiura, (3) Hookworm spp, (4)
Strongyloides stercoralis; (5) Enterobius vermicularis; (6) Species causing cutaneous larva
migrant; (7) Filaria spp; (8) Schistosoma spp; and (8) Taenia spp. For those species, the
fourth level of students competency has been formulated, in which students must be able to
Faculty of Medicine UNUD,MEU

51

Study Guide Infection and Infectious Diseases


perform clinical diagnosis based on physical examination and other additional examinations
(such as simple laboratory and X ray examination). Instead of just making clinical diagnosis,
the students should also be able to manage all related problems completely and individually.
The students must therefore be familiar with the life cycle, epidemiology, pathogenesis,
clinical manifestations, diagnosis as well as treatment procedures of all the above
mentioned helminthic infections, otherwise the four level of student competency can not be
achieved.
Learning Tasks
1. Differentiate the morphological characteristics of the class Nematode, Cestode and
Trematode
2. Describe their life cycles and identify each of their infective stages
3. Define their epidemiological standpoints and list the factors that are closely related
to the transmission of the infections
4. Identify their stages of development which are useful from the viewpoint of diagnosis
purpose and try to figure their morphological characteristics
5. Describe their pathogenesis and clinical manifestations
6. Manage appropriately the diagnosis, treatment, and prevention measures.
Self evaluation
Ni Made Artini, a previously healthy 26 year old female medical student of the Faculty of
Medicine University of Uadayana was admitted to the emergency room of Sanglah Hospital
due to seizure. She was born in the Village of Ketewel, Sub-district of Sukawati, district of
Gianyar. and attended primary and secondary high school at the same village. Her history
was otherwise unremarkable, although she reported that she had been suffering from
severe headache for a couple of months. Her neurological examination revealed no clear
abnormalities. However, a computerized tomography scan (CT scan) of the head showed
multiple calcified lesions in both cerebral hemispheres.
Questions
1. If it is a disease caused by parasitic infection, what parasite would you consider to
be responsible for the patients condition?
2. How does man acquire this infection?
3. What are the factors that may predispose to this infection?
4. How would you diagnose this patient properly?
5. How would you treat this patient appropriately?

Lecture 37
Filariasis
dr. K. Agus Somia, Sp.PD
Lymphatic filariasis is the commonest lymphatic system infection that is occurred in
community especially in eastern part of Indonesia. Lymphatic filariasis, onchocerciasis, and
loiasis are the three most important filarial infections of humans. Lymphatic filariasis is
caused by parasite transmitted by biting arthropods (mosquitoes). Almost 90% are caused
by Wuchereria bancrofti, whose only in human and most of the remainder are caused by
Brugia malayi. The major vectors for W. bancrofti are culicine mosquitoes in most urban and
semiurban areas, anopheline mosquitoes in the more rural areas of Africa and Aedes spp in
mnay of the endemic Pasific island.
A-35-year-old woman complaints her leg is swelling. The swollen is getting worst
and pain since 2 weeks ago. Half of his swollen leg looks redness and felt pain. The patient

Faculty of Medicine UNUD,MEU

52

Study Guide Infection and Infectious Diseases


lives in Bali and she comes from Kupang. She has been like this before and it is the 3 rd
times. She gets fevers, fatique, and headache.
Learning task:
1.
patient.
2.
3.
4.
5.

Define other things that should be found from history taking of this
Describe physical examination to diagnose this patient
Define other examination/ investigation to support the diagnosis
Define the invasive treatment should be done to this patient
Describe management/ treatment of this patient

Self assessment:
Choose RIGHT or WRONG of these statement
1. The possibly diagnosis is limfangitis
2. The most possibly diagnosis is filariasis with inflammation
3. It needs cell blood count and thick smear examination
4. Lymphograpy examination showed obstruction, atresia, or dilatation can
helps to diagnose this disease
5. Diethylcarbamazine 2-3 mg/kg, 3 times a day is given for 2-3 weeks.

Lecture 38
Dengue Viral Infection
dr. Made Susila Utama, Sp,PD
Abstract
Dengue fever/dengue hemorrhagic fever caused by dengue viruses (type 1,2,3 and
4), transmission from human to human is by the mosquito Aedes aegypti. Clinical spectrum
of dengue viral infections are wide variation, from undifferentiated fever, dengue fever,
dengue hemorrhagic fever and dengue shock syndrome. There is plasma leakage in
dengue hemorrhagic fever, so differentiated with dengue fever.
Dengue fever should be treated supportively. Dengue hemorrhagic fever/dengue
shock syndrome is life threatening and requires immediate evaluation of vital sign,
hemoconcentration, dehydration, urine output, electrolit imbalance.
Reference
Halstead SB. Dengue fever/ Dengue Haemorrhagic fever. Powderly WG.(Ed). Infectious
disease. Second ed. 2004. P. 1681-4
Dengue infection
Case:
A Male, 34 years old, Balinese, came to the Sanglah Hospital. The chief of complain was
fever since 2 days ago, he also complain about headache, joint pain, rash on the skin. The
neighbored was admitted in the hospital with DHF
Learning task:
1. What the some specific factor in the history and examination suggest the need for
making diagnose
2. what the laboratory examination the need for this patients
3. what the management for this patients

Faculty of Medicine UNUD,MEU

53

Study Guide Infection and Infectious Diseases


Self assessments
1. describe the clinical spectrum of dengue infection
1. describe the pathogenesis of DHF
2. describe the management approach for the dengue infections
References
1. Halstead SB. Dengue fever/ Dengue Haemorrhagic fever. Powderly WG.(Ed).
Infectious disease. Second ed. 2004. P. 1681-4

Lecture 39
Treatment of Helminthes Infection (PK/PD)
Dr.dr. B.K.Satriyasa,M.Rrepro
Anthelmintic
Helminthic infections still as a problem on the world. There are many of anthelminthic
drugs that can be used to eradicate the parasite in the intestinal tract or in the tissue of the
body. Most anthelminthics in use today are active against specific parasites, and some are
toxic. Therefore parasites must be identified before treatment is started. In this topic will be
introduced the drugs for anthelmintic so after this program all of student be able to choose
anthelmintic drugs for the patients in rationally.

Anthelmintic
1. Discuss anthelmintic drugs that use to eradicate or reduce the number of helminthic
parasites in the intestinal tract or tissue of the body
2. Discuss drugs of choice for the especially parasite and side effect of that drugs of
the body
3. Discuss the mechanism of action of anthelmintic drugs that you know
4. Discuss the principle of treatment of patient according to the parasite that will be
eradicated
Self assessment:
1. Drug of choice for Ascaris lumbricoides is:
a. Pyrantel pamoate
b. Albendazole
c. Piperazine
d. Levamisole
e. Praziquantel
2. Drug of choice for cutaneus larva migran is:
a. Pyrantel pamoate
b. Albendazole
c. Piperazine
d. Thiabendazole
e. Praziquantel
3. A patient suffered from taenia solium. Drug that can be used as drug of choice of this
worm is:
a. Pyrantel pamoate
b. Albendazole
c. Piperazine
d. Levamisole

Faculty of Medicine UNUD,MEU

54

Study Guide Infection and Infectious Diseases


e. Praziquantel

Lecture 40
Overview of Puerperal Infection
dr. Hariyasa Sanjaya,Sp.OG
1. Abstract:
Puerperal Infection is a general term used to describe any bacterial infection of
the genital tract after delivery along with preeclampsia and obstetrical
hemorrhage puerperal infection formed the lethal triad of causes of maternal
deaths because of effective antimicrobials, maternal deaths from infection have
become uncommon
2. Learning task:
2.1.
To understand definition of puerperial infection.
2.2.
To understand definition and management of puerperial fever
2.3.
To understand definition, predisposing factors, bacteriology and
management of uterine infection.
2.4.
To understand the complication of pelvic infection
2.5.
To understand the pathogenesis, clinical course and treatment of
infections of perineum, vagina and cervix.
2.6.
To understand the toxic shock syndrome
2.7.
To understand the prevention of puerperial infection
3. Case
A 25 year old woman (G1P1) presents to your clinic eight days postpartum,
complaining of a temperature of at least 38.5 degrees Celsius over the past 3
days, and a foul-smelling vaginal discharge. She is in otherwise good health,
and her baby, who was born by emergency Caesarian section in a rural clinic, is
doing well. Physical examination of your patient reveals an oral temperature of
38.6 degrees Celsius, a clean and non-weeping abdominal wound, and pain of
palpation of her uterus.
What is the differential diagnosis of the site of infection? What was most likely
the source of this infection? What features of your patients history and delivery
put her at higher risk for puerperal infection?
4. Self assessment:
1. How the student understand about definition, risk factors, pathogenesis,
complication and management of puerperial infection?
2. How the student understand the definition of puerperial fever and the
deferential diagnosis of puerperial fever?
3. How the student understand the prevention of puerperial fever?

Faculty of Medicine UNUD,MEU

55

Study Guide Infection and Infectious Diseases

Lecture 41
Overview of Sexual Transmitted Diseases
dr. A.A.G.P. Wiraguna, Sp.KK
Case 1:
A 27-year-old man had single painless ulcer on his glans penis 3 year ago. This ulcers
disappear without treatment. One year ago, he got married with a 23-year-old woman and
now his wife is pregnant for 4 months. His wife complain of having vaginal discharge with
itchy and odor. This man now has rash on whole body and mucopurulent urethral discharge,
they already went to a venereologist. The result of laboratories examinations shows VDRL
1:64 and doctor referred this couple to go to the Department Dermato-Venereology Sanglah
Hospital.
Learning task/ questions:
1. What other history you need to find out from these patients?
2. What laboratories examination needs to be done for this couple?
3. What is your diagnosis for this man?
4. What is your diagnosis for his wife?
5. What could possibly happen with her pregnancy?
6. What could possibly happen with their baby?
7. How would you treat this man, his wife, and their baby based on their conditions?
Self assessment:
1. Describe the stages clinical manifestation of syphilis.
2. Describe the causes of genital ulceration.
3. Describe microorganism pathologic of urethral discharge.
4. Describe the risk factors of sexually transmitted infection patient
5. How to prevent management of STI.
6. Describe microorganism pathologic of vaginal discharge and the clinical manifestation
7. How to treat clinical manifestation of vaginal discharge, the dose, and for how long.

Faculty of Medicine UNUD,MEU

56

Study Guide Infection and Infectious Diseases

~ CURRICULUM MAP ~
Smstr

Program or curriculum blocks

10

Senior Clerkship

Senior Clerkship

Senior clerkship

Medical
Emergency
(3 weeks)

Special Topic:
-Travel medicine
(2 weeks)

Elective Study III


(6 weeks)

Clinic Orientation
(Clerkship)
(6 weeks)

BCS (1 weeks)
The Respiratory
System and
Disorders
(4 weeks)

The
Cardiovascular
System and
Disorders
(4 weeks)

The Urinary
System and
Disorders
(3 weeks)

The Reproductive
System and
Disorders
(3 weeks)

BCS (1 weeks)
Alimentary
& hepatobiliary systems
& disorders
(4 Weeks)

BCS (1 weeks)
The Endocrine
System,
Metabolism and
Disorders
(4 weeks)

BCS (1 weeks)
Clinical Nutrition
and Disorders
(2 weeks)

BCS (1 weeks)

BCS (1 weeks)

Musculoskeletal
system &
connective
tissue disorders
(4 weeks)

Neuroscience
and
neurological
disorders
(4 weeks)

Behavior Change
and disorders
(4 weeks)

BCS (1 weeks)
Hematologic
system & disorders & clinical
oncology
(4 weeks)

BCS (1 weeks)
Immune
system &
disorders
(2 weeks)

BCS(1 weeks)
Infection
& infectious
diseases
(5 weeks)

BCS
(1 weeks)
The skin & hearing
system
& disorders
(3 weeks)

BCS (1 weeks)
Medical
Professionalism
(2 weeks)

BCS(1 weeks)
Evidence-based
Medical Practice
(2 weeks)

BCS (1 weeks)
Health Systembased Practice
(3 weeks)

BCS(1 weeks)
Community-based
practice
(4 weeks)

BCS (1 weeks)
Studium

Medical

BCS (1 weeks)
The cell

Growth

BCS (1 weeks)
Elective Study
II
(1 weeks)
5

Faculty of Medicine UNUD,MEU

BCS (1 weeks)

Special Topic :
- Palliative
medicine
-Compleme
ntary &
Alternative
Medicine
- Forensic
(3 weeks)

Elective
Study II
(1 weeks)

Special Topic
- Ergonomi
- Geriatri
(2 weeks)

Elective
Study I
(2 weeks)

The Visual
system &
disorders
(2 weeks)

57

Study Guide Infection and Infectious Diseases


Generale and
Humaniora
(3 weeks)

communication
(3 weeks)

as biochemical machinery
(3 weeks)

&
development
(4 weeks)

BCS (1 weeks)

BCS(1 weeks)

BCS: (1 weeks)

Pendidikan Pancasila & Kewarganegaraan (3 weeks)

REFFRENCES
1. Spicer WJ. (200): Clinical Bacteriology, Mycology, and Parasitology, An Illustrated
Colour Text. Churchill Livingstone, 14-19.
2. Clinical Bacteriology, Mycology and Parasitology : An Illustrated Colour Text. W.
John Spicer. Churchill-Livingstone
3. Brooks et al. pathogenesis and Control of Viral Diseases. In: Lange Medical
Microbiology. 23rd ed. McGraw Hill. International Ed. 2004. p. 394 413.(Principles
of Viral Infection)
4. Levinson et al. Lange Medical Microbiology & immunology. Examination & Board
review. 8th ed. McGraw Hill. International Ed. 2004. p. 186 220, 259-269, 244-250.
(Principles of Viral Infection)
5. Roitt. I., Brostoff.J., Male. D. Immunology
6. Durack DT, Whitley RJ, and Scheld WM. Introduction: Approach to the Patient with
Central nervous System Infection. In : Scheld WM, Whitley RJ, Durack DT, (eds).
Infections of The Central Nervous System. Raven Press. New York. 1991 p. 1-4.
7. Victor M and ropper AH. Infections of the Nervous System (Bacterial, Fungal,
spirochetal, Parasitic) and Sarcoid. In: Adams and Victors principles of the
Neurology. 7th ed. McGraw-Hill. New York/Toronto. P. 734-780.
8. Ottesen EA. Filariasis.in Powderly WG. (ed). Infectious Diseases. 2nd ed. P.1607-13.
9. Ringsrud KM, Linne JJ. Urinalysis and Body Fluids A Colortext and Atlas. 1st ed.
Mosby. St. Louis/ Toronto. 1995. p. 95-206.
10. Burtis CA. Tietz Fundamentals of Clinical Chemistry. 4th ed. WB Saunders Company.
Philadelphia/ Tokyo. 1996. p. 558-561.
11. Simmons A. Statland BE. Hematology A combined Theoritical and technical
Approach. 2nd ed. Buuterworth-Heinemann. Boston/ Singapore. 1997. p. 129-142.
12. Stites DP, Terr AI, Parslow TG. Medical Immunology. 9th ed. Prentice-Hall
International. 1997. p. 264-269.
13. Kasper DL, Fauci AS, Longo DL, Braunwald E, et al. Harrisons Principles of Internal
Medicine. 16th ed. Vol 1. McGraw-Hill. New York/ Toronto. 2005. p. 981-1103.
14. Sutton D. Radiology and Imaging for Medical Students. Churchill Livingstone. 7 th ed.
1998.
15. Grainger RG and Allison DJ. Diagnostic Radiology. Churchill Livingstone. 2nd ed.
1993.
16. McAdam AJ and Kumar S. Infectious Diseases in Kumar V, Contran RS and Robbins
SL, Robbins Basic Pathology. P. 344-398.
17. Andrews. Diseases of The Skin. 9th ed.
18. Bryceson A. Leprosy. 3rd ed.
19. King & Nicole. Sexually Transmitted Diseases. 2003
20. Holmes KK, Spiring PF, Mirdh P. Sexually Transmitted Diseases. 3rd ed. McGrawHill. 1999.
21. McMillan A, Young H, Ogilvie MM, Scott GR. Clinical Practice in Sexually
Transmissible Infection. Saunders. 2002.
22. Braunwalds Heart Disease. Subacute bacterial endocarditis.

Faculty of Medicine UNUD,MEU

58

Study Guide Infection and Infectious Diseases

Faculty of Medicine UNUD,MEU

59

You might also like